first commit

This commit is contained in:
François Pelletier 2020-01-02 17:35:21 -05:00
commit f6734c7548
33 changed files with 3480 additions and 0 deletions

279
.gitignore vendored Normal file
View file

@ -0,0 +1,279 @@
## Core latex/pdflatex auxiliary files:
*.aux
*.lof
*.log
*.lot
*.fls
*.out
*.toc
*.fmt
*.fot
*.cb
*.cb2
.*.lb
## Intermediate documents:
*.dvi
*.xdv
*-converted-to.*
# these rules might exclude image files for figures etc.
# *.ps
# *.eps
# *.pdf
## Generated if empty string is given at "Please type another file name for output:"
.pdf
## Bibliography auxiliary files (bibtex/biblatex/biber):
*.bbl
*.bcf
*.blg
*-blx.aux
*-blx.bib
*.run.xml
## Build tool auxiliary files:
*.fdb_latexmk
*.synctex
*.synctex(busy)
*.synctex.gz
*.synctex.gz(busy)
*.pdfsync
## Build tool directories for auxiliary files
# latexrun
latex.out/
## Auxiliary and intermediate files from other packages:
# algorithms
*.alg
*.loa
# achemso
acs-*.bib
# amsthm
*.thm
# beamer
*.nav
*.pre
*.snm
*.vrb
# changes
*.soc
# comment
*.cut
# cprotect
*.cpt
# elsarticle (documentclass of Elsevier journals)
*.spl
# endnotes
*.ent
# fixme
*.lox
# feynmf/feynmp
*.mf
*.mp
*.t[1-9]
*.t[1-9][0-9]
*.tfm
#(r)(e)ledmac/(r)(e)ledpar
*.end
*.?end
*.[1-9]
*.[1-9][0-9]
*.[1-9][0-9][0-9]
*.[1-9]R
*.[1-9][0-9]R
*.[1-9][0-9][0-9]R
*.eledsec[1-9]
*.eledsec[1-9]R
*.eledsec[1-9][0-9]
*.eledsec[1-9][0-9]R
*.eledsec[1-9][0-9][0-9]
*.eledsec[1-9][0-9][0-9]R
# glossaries
*.acn
*.acr
*.glg
*.glo
*.gls
*.glsdefs
*.lzo
*.lzs
# uncomment this for glossaries-extra (will ignore makeindex's style files!)
# *.ist
# gnuplottex
*-gnuplottex-*
# gregoriotex
*.gaux
*.gtex
# htlatex
*.4ct
*.4tc
*.idv
*.lg
*.trc
*.xref
# hyperref
*.brf
# knitr
*-concordance.tex
# TODO Comment the next line if you want to keep your tikz graphics files
*.tikz
*-tikzDictionary
# listings
*.lol
# luatexja-ruby
*.ltjruby
# makeidx
*.idx
*.ilg
*.ind
# minitoc
*.maf
*.mlf
*.mlt
*.mtc[0-9]*
*.slf[0-9]*
*.slt[0-9]*
*.stc[0-9]*
# minted
_minted*
*.pyg
# morewrites
*.mw
# nomencl
*.nlg
*.nlo
*.nls
# pax
*.pax
# pdfpcnotes
*.pdfpc
# sagetex
*.sagetex.sage
*.sagetex.py
*.sagetex.scmd
# scrwfile
*.wrt
# sympy
*.sout
*.sympy
sympy-plots-for-*.tex/
# pdfcomment
*.upa
*.upb
# pythontex
*.pytxcode
pythontex-files-*/
# tcolorbox
*.listing
# thmtools
*.loe
# TikZ & PGF
*.dpth
*.md5
*.auxlock
# todonotes
*.tdo
# vhistory
*.hst
*.ver
# easy-todo
*.lod
# xcolor
*.xcp
# xmpincl
*.xmpi
# xindy
*.xdy
# xypic precompiled matrices and outlines
*.xyc
*.xyd
# endfloat
*.ttt
*.fff
# Latexian
TSWLatexianTemp*
## Editors:
# WinEdt
*.bak
*.sav
# Texpad
.texpadtmp
# LyX
*.lyx~
# Kile
*.backup
# gummi
.*.swp
# KBibTeX
*~[0-9]*
# TeXnicCenter
*.tps
# auto folder when using emacs and auctex
./auto/*
*.el
# expex forward references with \gathertags
*-tags.tex
# standalone packages
*.sta
# Makeindex log files
*.lpz
.Rproj.user
*.Rproj

21
BoA.csv Normal file
View file

@ -0,0 +1,21 @@
Date AdjClose
2013-09-30 13.90
2013-09-23 13.90
2013-09-16 14.44
2013-09-09 14.49
2013-09-03 14.36
2013-08-26 14.11
2013-08-19 14.56
2013-08-12 14.41
2013-08-05 14.44
2013-07-29 14.83
2013-07-22 14.72
2013-07-15 14.74
2013-07-08 13.77
2013-07-01 13.05
2013-06-24 12.85
2013-06-17 12.68
2013-06-10 13.06
2013-06-03 13.37
2013-05-28 13.64
2013-05-20 13.22
1 Date AdjClose
2 2013-09-30 13.90
3 2013-09-23 13.90
4 2013-09-16 14.44
5 2013-09-09 14.49
6 2013-09-03 14.36
7 2013-08-26 14.11
8 2013-08-19 14.56
9 2013-08-12 14.41
10 2013-08-05 14.44
11 2013-07-29 14.83
12 2013-07-22 14.72
13 2013-07-15 14.74
14 2013-07-08 13.77
15 2013-07-01 13.05
16 2013-06-24 12.85
17 2013-06-17 12.68
18 2013-06-10 13.06
19 2013-06-03 13.37
20 2013-05-28 13.64
21 2013-05-20 13.22

5
CHANGES Normal file
View file

@ -0,0 +1,5 @@
7 octobre 2013:
- J'ai corrigé l'énoncé de l'exercice Option de vente (Calculatrice) pour qu'il soit plus précis
- J'ai ajouté un template LaTeX pour créer les futurs documents
- J'ai créé un document d'introduction à Sweave que je vais développer au fil du temps
- J'ai modifié l'éconcé de l'exercice Bank of America et ajouté le corrigé

20
LICENSE Normal file
View file

@ -0,0 +1,20 @@
Paternité - Partage des Conditions Initiales à l'Identique 2.5 Canada (CC BY-SA 2.5 CA)
Vous êtes libres:
de reproduire, distribuer et communiquer cette création au public
de modifier cette création
d'utiliser cette création à des fins commerciales
Selon les conditions suivantes :
Paternité — Vous devez citer le nom de l'auteur original.
Partage des Conditions Initiales à l'Identique — Si vous modifiez, transformez ou adaptez cette création, vous n'avez le droit de distribuer la création qui en résulte que sous un contrat identique à celui-ci.
comprenant bien que :
Renonciation — Nimporte laquelle des conditions ci-dessus peut être levée si vous avez lautorisation du titulaire de droits.
Public Domain — Là où lœuvre ou un quelconque de ses éléments est dans le domaine public selon le droit applicable, ce statut nest en aucune façon affecté par la licence.
Autres droits — Les droits suivants ne sont en aucune manière affectés par la licence :
Vos prérogatives issues des exceptions et limitations aux droits exclusifs ou fair use;
Les droits moraux de lauteur;
Droits quautrui peut avoir soit sur lœuvre elle-même soit sur la façon dont elle est utilisée, comme le droit à limage ou les droits à la vie privée.
Remarque — A chaque réutilisation ou distribution, vous devez faire apparaître clairement aux autres les conditions contractuelles de mise à disposition de cette création.

BIN
Lissage.Exponentiel.I.ods Normal file

Binary file not shown.

42
Makefile Normal file
View file

@ -0,0 +1,42 @@
# This makefile provides three targets: tex, pdf and clean.
# The default is "pdf".
# To make a tex file type "make tex"
# To make a pdf file, type "make pdf" or simply "make".
# To remove all files generated by make, type "make clean".
#
# Rouben Rostmaian, minor adjustments for Rnw Nicholas Lewin-koh
# September 2002 March 2005
# Further adjustments Vincent Goulet
# April 2008
# Minor adjustments for Linux François Pelletier
# December 2013
MASTER = exercices_series_chrono.pdf
# The master document depends on all TeX files
RNWFILES = $(wildcard *.Rnw)
TEXFILES = $(wildcard *.tex)
# The work horses
SWEAVE = R CMD Sweave --encoding=utf8
TEXI2DVI = texi2dvi -b --pdf
RM = rm -rf
.PHONY: tex pdf clean
pdf: $(MASTER)
tex: $(RNWFILES:.Rnw=.tex)
%.tex: %.Rnw
$(SWEAVE) '$<'
$(MASTER): $(RNWFILES) $(TEXFILES)
$(TEXI2DVI) $(MASTER:.pdf=.tex)
clean:
$(RM) $(RNWFILES:.Rnw=.tex) \
*-[0-9][0-9][0-9].eps \
*-[0-9][0-9][0-9].pdf \
*.aux *.log *.blg *.bbl *.out *.rel *.t2d *.toc *~ Rplots.ps Rplots.pdf
$(RM) exercice1-*.pdf serie*.tex

8
README.md Normal file
View file

@ -0,0 +1,8 @@
ACT-2010-Exercices
==================
Exercices pour le cours ACT-2010 de l'École d'actuariat de l'Université Laval, Québec
Pour de l'aide sur le site github et sur git en général, je vous suggère cette page:
http://www-cs-students.stanford.edu/~blynn/gitmagic/intl/fr/

43
Sweave.sty Normal file
View file

@ -0,0 +1,43 @@
\NeedsTeXFormat{LaTeX2e}
\ProvidesPackage{Sweave}{}
\RequirePackage{ifthen}
\newboolean{Sweave@gin}
\setboolean{Sweave@gin}{true}
\newboolean{Sweave@ae}
\setboolean{Sweave@ae}{true}
\DeclareOption{nogin}{\setboolean{Sweave@gin}{false}}
\DeclareOption{noae}{\setboolean{Sweave@ae}{false}}
\ProcessOptions
\RequirePackage{graphicx,fancyvrb}
\IfFileExists{upquote.sty}{\RequirePackage{upquote}}{}
\ifthenelse{\boolean{Sweave@gin}}{\setkeys{Gin}{width=0.8\textwidth}}{}%
\ifthenelse{\boolean{Sweave@ae}}{%
\RequirePackage[T1]{fontenc}
\RequirePackage{ae}
}{}%
\DefineVerbatimEnvironment{Sinput}{Verbatim}{fontshape=sl}
\DefineVerbatimEnvironment{Soutput}{Verbatim}{}
\DefineVerbatimEnvironment{Scode}{Verbatim}{fontshape=sl}
\ifdefined\Schunk%
\message{\string Environment Schunk is already defined, stay with former definition}%
\else
\newenvironment{Schunk}{}{}%
\fi
\newcommand{\Sconcordance}[1]{%
\ifx\pdfoutput\undefined%
\csname newcount\endcsname\pdfoutput\fi%
\ifcase\pdfoutput\special{#1}%
\else%
\begingroup%
\pdfcompresslevel=0%
\immediate\pdfobj stream{#1}%
\pdfcatalog{/SweaveConcordance \the\pdflastobj\space 0 R}%
\endgroup%
\fi}

View file

@ -0,0 +1,10 @@
An;TauxEmploiPlus55
2003;24.3
2004;25.4
2005;26.0
2006;26.4
2007;27.4
2008;27.8
2009;28.5
2010;29.2
2011;29.6
1 An TauxEmploiPlus55
2 2003 24.3
3 2004 25.4
4 2005 26.0
5 2006 26.4
6 2007 27.4
7 2008 27.8
8 2009 28.5
9 2010 29.2
10 2011 29.6

264
blackberry.csv Normal file
View file

@ -0,0 +1,264 @@
Date;Open;High;Low;Close;Volume;Adj Close
2013-09-20;10.50;10.51;8.01;8.73;72072500;8.73
2013-09-19;10.48;10.68;10.23;10.52;26905900;10.52
2013-09-18;10.69;10.79;10.24;10.40;21009500;10.40
2013-09-17;10.31;10.78;10.31;10.55;19700000;10.55
2013-09-16;10.33;10.44;10.20;10.31;13960900;10.31
2013-09-13;10.45;10.48;10.24;10.28;17351900;10.28
2013-09-12;10.52;10.69;10.35;10.43;17925100;10.43
2013-09-11;10.82;10.85;10.37;10.43;27800700;10.43
2013-09-10;11.16;11.20;10.89;10.94;34307300;10.94
2013-09-09;11.15;11.65;11.03;11.53;36864900;11.53
2013-09-06;11.14;11.15;10.81;10.84;16919500;10.84
2013-09-05;11.15;11.33;10.77;10.99;40031400;10.99
2013-09-04;10.38;10.89;10.21;10.75;30307900;10.75
2013-09-03;10.32;10.48;10.14;10.21;22668700;10.21
2013-08-30;10.17;10.26;10.06;10.12;12720300;10.12
2013-08-29;10.35;10.45;10.20;10.26;9943900;10.26
2013-08-28;10.00;10.68;9.47;10.30;31274900;10.30
2013-08-27;10.34;10.39;10.01;10.03;20685200;10.03
2013-08-26;10.32;10.64;10.31;10.42;14351000;10.42
2013-08-23;10.24;10.43;10.19;10.34;11328300;10.34
2013-08-22;10.12;10.41;10.12;10.19;8889500;10.19
2013-08-21;10.39;10.50;10.12;10.25;22112800;10.25
2013-08-20;10.18;10.54;10.05;10.54;16291600;10.54
2013-08-19;10.29;10.50;10.09;10.32;17854700;10.32
2013-08-16;10.80;10.90;10.40;10.51;29248600;10.51
2013-08-15;10.90;11.00;10.74;10.96;20780600;10.96
2013-08-14;11.18;11.56;10.77;11.04;44830500;11.04
2013-08-13;10.99;12.18;10.74;10.93;127211600;10.93
2013-08-12;10.65;10.93;10.12;10.78;92858900;10.78
2013-08-09;9.78;9.98;9.54;9.76;51295700;9.76
2013-08-08;9.23;9.40;9.15;9.23;8858400;9.23
2013-08-07;9.49;9.64;9.11;9.21;17327300;9.21
2013-08-06;9.64;9.88;9.41;9.57;24104900;9.57
2013-08-05;9.00;9.77;8.98;9.58;33620900;9.58
2013-08-02;8.77;8.94;8.72;8.92;8390200;8.92
2013-08-01;8.89;8.91;8.72;8.76;8673400;8.76
2013-07-31;8.65;8.91;8.65;8.84;13802700;8.84
2013-07-30;8.64;8.92;8.62;8.70;14920400;8.70
2013-07-29;8.80;8.90;8.57;8.67;17088500;8.67
2013-07-26;8.94;9.00;8.77;8.88;13484200;8.88
2013-07-25;8.95;9.08;8.90;8.98;11807800;8.98
2013-07-24;9.01;9.09;8.93;9.00;13026500;9.00
2013-07-23;9.14;9.18;8.95;9.02;13123600;9.02
2013-07-22;8.92;9.12;8.90;9.08;15415300;9.08
2013-07-19;9.24;9.28;8.90;8.97;23398400;8.97
2013-07-18;9.23;9.31;9.07;9.12;12500000;9.12
2013-07-17;9.07;9.32;9.07;9.24;15088100;9.24
2013-07-16;9.36;9.37;9.10;9.13;18645600;9.13
2013-07-15;9.20;9.55;9.18;9.37;17644700;9.37
2013-07-12;9.29;9.33;9.20;9.24;12874700;9.24
2013-07-11;9.28;9.35;9.16;9.33;17583600;9.33
2013-07-10;9.74;9.74;9.10;9.28;31079300;9.28
2013-07-09;9.61;9.93;9.55;9.64;29795600;9.64
2013-07-08;9.61;9.67;9.48;9.55;16762300;9.55
2013-07-05;9.65;9.88;9.50;9.55;19190100;9.55
2013-07-03;9.51;9.74;9.42;9.65;22608200;9.65
2013-07-02;10.20;10.27;9.63;9.70;44561500;9.70
2013-07-01;10.03;10.62;9.96;10.29;41076400;10.29
2013-06-28;10.71;10.98;10.25;10.46;136025600;10.46
2013-06-27;15.04;15.09;14.28;14.48;37407900;14.48
2013-06-26;14.70;15.00;14.70;14.91;23230800;14.91
2013-06-25;14.39;14.62;14.22;14.56;17641400;14.56
2013-06-24;13.50;14.29;13.50;14.10;19076300;14.10
2013-06-21;13.97;14.05;13.68;13.78;14323900;13.78
2013-06-20;13.92;14.18;13.82;13.98;16806700;13.98
2013-06-19;14.44;14.61;13.95;14.18;34716900;14.18
2013-06-18;14.32;15.00;14.26;14.84;31795000;14.84
2013-06-17;14.60;14.65;14.18;14.30;12316700;14.30
2013-06-14;14.64;14.93;14.35;14.44;24987700;14.44
2013-06-13;13.95;14.49;13.83;14.42;32443000;14.42
2013-06-12;13.67;13.79;13.50;13.56;9469200;13.56
2013-06-11;13.82;13.95;13.63;13.66;11907100;13.66
2013-06-10;13.87;14.09;13.70;13.95;16515300;13.95
2013-06-07;13.82;14.13;13.74;13.86;18496900;13.86
2013-06-06;13.56;13.90;13.53;13.71;15276600;13.71
2013-06-05;13.49;13.92;13.37;13.63;24596400;13.63
2013-06-04;13.57;13.90;13.48;13.54;15562300;13.54
2013-06-03;14.03;14.05;13.30;13.61;25458800;13.61
2013-05-31;14.50;14.52;13.83;13.96;30919100;13.96
2013-05-30;14.26;14.56;14.06;14.42;20257500;14.42
2013-05-29;14.30;14.54;14.16;14.17;17250300;14.17
2013-05-28;14.53;14.61;14.31;14.36;14657900;14.36
2013-05-24;14.46;14.65;14.36;14.48;13456300;14.48
2013-05-23;14.12;14.67;14.08;14.43;15343200;14.43
2013-05-22;14.60;14.84;14.34;14.46;20391200;14.46
2013-05-21;15.01;15.04;14.65;14.77;14376000;14.77
2013-05-20;14.70;15.27;14.53;14.98;18726400;14.98
2013-05-17;14.71;14.86;14.52;14.62;15557100;14.62
2013-05-16;15.07;15.13;14.60;14.69;20357600;14.69
2013-05-15;14.90;15.42;14.72;15.00;26858700;15.00
2013-05-14;16.01;16.14;14.92;15.25;52632100;15.25
2013-05-13;15.73;16.16;15.70;15.88;29220200;15.88
2013-05-10;15.49;15.68;15.31;15.54;17895600;15.54
2013-05-09;15.15;15.68;14.87;15.39;40657600;15.39
2013-05-08;15.10;15.15;14.80;14.90;17680100;14.90
2013-05-07;15.39;15.51;14.68;14.82;35314700;14.82
2013-05-06;15.66;15.98;15.45;15.57;23381500;15.57
2013-05-03;16.08;16.11;15.59;15.63;24469800;15.63
2013-05-02;15.65;16.06;15.30;15.70;26204800;15.70
2013-05-01;16.37;16.49;15.46;15.80;33440200;15.80
2013-04-30;15.79;16.59;15.75;16.29;42800400;16.29
2013-04-29;15.41;15.68;15.27;15.61;26672000;15.61
2013-04-26;14.84;15.09;14.66;15.02;17746700;15.02
2013-04-25;15.08;15.39;14.63;14.81;30676900;14.81
2013-04-24;14.48;14.96;14.38;14.90;24928000;14.90
2013-04-23;14.08;14.54;13.96;14.33;20400500;14.33
2013-04-22;13.85;14.13;13.61;13.99;19243400;13.99
2013-04-19;13.50;13.91;13.10;13.84;24013200;13.84
2013-04-18;13.72;14.04;13.38;13.50;26546600;13.50
2013-04-17;13.88;14.20;13.72;13.91;29646100;13.91
2013-04-16;13.97;14.15;13.85;13.92;19340500;13.92
2013-04-15;13.45;13.85;13.42;13.71;21250400;13.71
2013-04-12;13.67;14.04;13.38;13.64;34977000;13.64
2013-04-11;14.16;14.20;13.28;13.55;79542900;13.55
2013-04-10;14.86;14.99;14.47;14.69;36632400;14.69
2013-04-09;14.74;14.92;14.61;14.85;19527700;14.85
2013-04-08;14.89;15.06;14.61;14.67;27616900;14.67
2013-04-05;14.76;14.94;14.56;14.70;20961200;14.70
2013-04-04;15.09;15.31;14.92;14.99;27401500;14.99
2013-04-03;15.00;15.25;14.61;15.03;39206200;15.03
2013-04-02;15.20;15.47;15.04;15.14;31329600;15.14
2013-04-01;14.66;15.36;14.29;15.11;57405400;15.11
2013-03-28;15.18;15.55;14.14;14.45;107118000;14.45
2013-03-27;14.57;14.92;14.30;14.57;50966700;14.57
2013-03-26;14.57;14.73;14.25;14.46;36076800;14.46
2013-03-25;13.97;14.62;13.64;14.23;78015800;14.23
2013-03-22;16.43;16.82;14.64;14.91;123949500;14.91
2013-03-21;15.97;16.48;15.95;16.16;46990300;16.16
2013-03-20;15.95;16.44;15.67;16.00;84106500;16.00
2013-03-19;15.12;15.30;14.86;15.03;27680400;15.03
2013-03-18;14.79;15.59;14.65;15.02;51108500;15.02
2013-03-15;15.14;15.27;14.80;14.98;39219000;14.98
2013-03-14;15.82;16.01;15.04;15.06;96029000;15.06
2013-03-13;14.17;15.70;14.11;15.65;82044100;15.65
2013-03-12;15.22;15.39;14.04;14.47;84606100;14.47
2013-03-11;13.13;14.92;13.00;14.90;92815900;14.90
2013-03-08;13.53;13.53;13.02;13.06;21945800;13.06
2013-03-07;13.46;13.55;12.98;13.22;27168300;13.22
2013-03-06;12.66;13.56;12.56;13.35;49755000;13.35
2013-03-05;12.86;12.92;12.55;12.58;27086200;12.58
2013-03-04;13.20;13.28;12.76;12.81;26847900;12.81
2013-03-01;13.44;13.78;13.03;13.26;39291800;13.26
2013-02-28;13.15;13.79;12.94;13.36;43004000;13.36
2013-02-27;12.93;13.68;12.75;13.25;38537000;13.25
2013-02-26;13.34;13.45;12.56;13.06;51689800;13.06
2013-02-25;13.66;13.76;13.05;13.25;35725200;13.25
2013-02-22;13.81;13.98;13.04;13.18;42572100;13.18
2013-02-21;13.94;14.05;13.35;13.88;49801500;13.88
2013-02-20;13.99;14.38;13.65;13.71;55153100;13.71
2013-02-19;14.25;14.82;14.17;14.36;46121000;14.36
2013-02-15;15.30;15.49;13.96;14.16;92805500;14.16
2013-02-14;13.20;15.16;12.94;15.07;112596700;15.07
2013-02-13;15.10;15.39;13.81;13.99;77515100;13.99
2013-02-12;15.47;16.07;15.15;15.20;44203400;15.20
2013-02-11;15.72;16.18;15.29;15.73;60717500;15.73
2013-02-08;17.15;17.22;16.44;16.49;61324600;16.49
2013-02-07;16.68;16.98;16.20;16.96;75845700;16.96
2013-02-06;15.52;16.54;15.28;16.05;100512100;16.05
2013-02-05;15.79;16.89;15.53;16.02;127576800;16.02
2013-02-04;13.71;15.01;13.52;14.98;82670900;14.98
2013-02-01;13.45;13.55;12.15;13.03;59354800;13.03
2013-01-31;12.45;13.35;12.15;12.98;111855800;12.98
2013-01-30;16.08;16.62;13.72;13.78;224097000;13.78
2013-01-29;15.92;16.00;14.68;15.66;106906200;15.66
2013-01-28;17.82;17.89;16.03;16.18;88895700;16.18
2013-01-25;18.09;18.24;17.40;17.54;57585500;17.54
2013-01-24;16.58;18.32;16.33;17.74;107804000;17.74
2013-01-23;17.87;18.08;17.16;17.35;75614300;17.35
2013-01-22;17.07;17.90;16.90;17.90;110834000;17.90
2013-01-18;15.86;15.94;15.34;15.84;71198300;15.84
2013-01-17;14.89;14.92;14.46;14.91;38902900;14.91
2013-01-16;14.75;15.36;14.65;14.74;67643400;14.74
2013-01-15;14.90;15.06;14.24;14.48;59086400;14.48
2013-01-14;13.95;15.09;13.90;14.95;143607300;14.95
2013-01-11;11.73;13.60;11.68;13.56;111026800;13.56
2013-01-10;11.85;11.98;11.61;11.92;24542500;11.92
2013-01-09;11.94;12.02;11.46;11.48;23609900;11.48
2013-01-08;11.99;12.24;11.74;11.91;33655500;11.91
2013-01-07;12.08;12.14;11.72;11.95;21292900;11.95
2013-01-04;11.64;12.03;11.44;11.95;30667000;11.95
2013-01-03;11.80;11.84;11.40;11.47;25866100;11.47
2013-01-02;12.32;12.37;11.64;11.72;29400600;11.72
2012-12-31;11.83;11.97;11.67;11.87;25606900;11.87
2012-12-28;11.70;12.10;11.57;11.79;28917100;11.79
2012-12-27;12.06;12.16;11.50;11.76;51026900;11.76
2012-12-26;10.79;12.03;10.66;11.83;70807700;11.83
2012-12-24;10.72;10.98;10.59;10.61;28912100;10.61
2012-12-21;11.84;12.39;10.91;10.91;149196000;10.91
2012-12-20;13.64;14.13;13.53;14.12;49596800;14.12
2012-12-19;13.55;13.89;13.52;13.63;26795900;13.63
2012-12-18;13.75;13.88;13.28;13.72;39727800;13.72
2012-12-17;14.15;14.20;13.66;13.93;44063800;13.93
2012-12-14;13.86;14.21;13.53;14.04;53970900;14.04
2012-12-13;13.45;14.16;13.42;13.86;82171300;13.86
2012-12-12;12.61;13.34;12.36;13.31;63731100;13.31
2012-12-11;11.96;12.67;11.88;12.60;43231600;12.60
2012-12-10;11.98;12.12;11.66;11.88;24329200;11.88
2012-12-07;11.85;12.13;11.75;12.01;24527000;12.01
2012-12-06;12.01;12.18;11.62;11.89;33156700;11.89
2012-12-05;11.62;12.12;11.47;11.94;32036200;11.94
2012-12-04;11.59;11.73;11.44;11.56;21189500;11.56
2012-12-03;11.39;11.82;11.25;11.60;38289200;11.60
2012-11-30;11.65;11.84;11.38;11.60;35486800;11.60
2012-11-29;12.28;12.30;11.37;11.54;85005500;11.54
2012-11-28;10.31;11.26;10.20;11.10;53958900;11.10
2012-11-27;11.98;12.00;10.71;10.72;84235200;10.72
2012-11-26;11.96;12.14;11.44;11.98;87527400;11.98
2012-11-23;11.60;11.86;11.26;11.66;74491400;11.66
2012-11-21;9.66;10.35;9.62;10.26;39520700;10.26
2012-11-20;10.02;10.04;9.57;9.71;49958400;9.71
2012-11-19;9.15;9.80;9.03;9.59;42524400;9.59
2012-11-16;8.85;9.30;8.53;9.20;35216100;9.20
2012-11-15;8.47;8.84;8.47;8.79;19957600;8.79
2012-11-14;8.52;8.67;8.38;8.49;15260800;8.49
2012-11-13;8.80;8.85;8.31;8.40;22877800;8.40
2012-11-12;9.05;9.07;8.67;8.81;20674900;8.81
2012-11-09;8.20;8.70;8.20;8.54;20708700;8.54
2012-11-08;8.46;8.52;8.14;8.20;16511200;8.20
2012-11-07;9.12;9.15;8.23;8.24;35905800;8.24
2012-11-06;9.01;9.08;8.87;9.06;18275700;9.06
2012-11-05;8.80;9.17;8.74;8.90;28343600;8.90
2012-11-02;8.84;8.91;8.69;8.71;24117600;8.71
2012-11-01;8.06;8.85;8.01;8.70;39524200;8.70
2012-10-31;7.59;8.00;7.53;7.93;13113900;7.93
2012-10-26;7.70;7.79;7.51;7.57;11489100;7.57
2012-10-25;7.64;7.77;7.53;7.71;10854300;7.71
2012-10-24;7.68;7.69;7.50;7.52;9163800;7.52
2012-10-23;7.59;7.90;7.46;7.62;15294900;7.62
2012-10-22;7.78;7.84;7.56;7.69;11932900;7.69
2012-10-19;7.88;7.90;7.64;7.76;19095100;7.76
2012-10-18;7.99;8.07;7.83;7.84;16355600;7.84
2012-10-17;7.80;8.08;7.78;7.99;14365900;7.99
2012-10-16;7.82;7.88;7.72;7.83;11589300;7.83
2012-10-15;7.83;7.90;7.71;7.80;9162500;7.80
2012-10-12;7.88;8.03;7.72;7.80;13505600;7.80
2012-10-11;7.82;8.00;7.77;7.91;11652000;7.91
2012-10-10;7.76;7.86;7.58;7.76;19016500;7.76
2012-10-09;8.06;8.11;7.77;7.80;23247400;7.80
2012-10-08;8.22;8.45;8.10;8.24;12219700;8.24
2012-10-05;8.35;8.44;8.18;8.22;15775700;8.22
2012-10-04;8.13;8.27;8.04;8.23;19958600;8.23
2012-10-03;8.35;8.35;8.00;8.06;27573900;8.06
2012-10-02;7.90;8.44;7.83;8.27;55091100;8.27
2012-10-01;7.62;7.88;7.27;7.86;52534300;7.86
2012-09-28;8.20;8.20;7.46;7.50;139438500;7.50
2012-09-27;7.20;7.23;6.75;7.14;71975800;7.14
2012-09-26;6.84;7.09;6.63;7.00;43862900;7.00
2012-09-25;6.37;6.71;6.25;6.60;45954900;6.60
2012-09-24;6.37;6.45;6.22;6.31;20380000;6.31
2012-09-21;6.80;6.84;6.43;6.46;47179100;6.46
2012-09-20;7.11;7.22;6.90;6.91;14097400;6.91
2012-09-19;7.48;7.48;7.18;7.21;15151800;7.21
2012-09-18;7.17;7.81;7.15;7.42;35720600;7.42
2012-09-17;7.39;7.46;7.18;7.25;14519100;7.25
2012-09-14;7.44;7.86;7.43;7.56;21556300;7.56
2012-09-13;7.38;7.59;7.30;7.43;14791300;7.43
2012-09-12;7.54;7.60;7.24;7.42;14702500;7.42
2012-09-11;7.18;7.55;7.12;7.46;17342600;7.46
2012-09-10;7.14;7.25;7.00;7.15;14868100;7.15
2012-09-07;6.73;7.22;6.67;7.19;23231000;7.19
2012-09-06;6.59;6.80;6.33;6.72;22312100;6.72
2012-09-05;6.61;6.75;6.38;6.52;7858000;6.52
2012-09-04;6.66;6.69;6.57;6.65;9693500;6.65
1 Date Open High Low Close Volume Adj Close
2 2013-09-20 10.50 10.51 8.01 8.73 72072500 8.73
3 2013-09-19 10.48 10.68 10.23 10.52 26905900 10.52
4 2013-09-18 10.69 10.79 10.24 10.40 21009500 10.40
5 2013-09-17 10.31 10.78 10.31 10.55 19700000 10.55
6 2013-09-16 10.33 10.44 10.20 10.31 13960900 10.31
7 2013-09-13 10.45 10.48 10.24 10.28 17351900 10.28
8 2013-09-12 10.52 10.69 10.35 10.43 17925100 10.43
9 2013-09-11 10.82 10.85 10.37 10.43 27800700 10.43
10 2013-09-10 11.16 11.20 10.89 10.94 34307300 10.94
11 2013-09-09 11.15 11.65 11.03 11.53 36864900 11.53
12 2013-09-06 11.14 11.15 10.81 10.84 16919500 10.84
13 2013-09-05 11.15 11.33 10.77 10.99 40031400 10.99
14 2013-09-04 10.38 10.89 10.21 10.75 30307900 10.75
15 2013-09-03 10.32 10.48 10.14 10.21 22668700 10.21
16 2013-08-30 10.17 10.26 10.06 10.12 12720300 10.12
17 2013-08-29 10.35 10.45 10.20 10.26 9943900 10.26
18 2013-08-28 10.00 10.68 9.47 10.30 31274900 10.30
19 2013-08-27 10.34 10.39 10.01 10.03 20685200 10.03
20 2013-08-26 10.32 10.64 10.31 10.42 14351000 10.42
21 2013-08-23 10.24 10.43 10.19 10.34 11328300 10.34
22 2013-08-22 10.12 10.41 10.12 10.19 8889500 10.19
23 2013-08-21 10.39 10.50 10.12 10.25 22112800 10.25
24 2013-08-20 10.18 10.54 10.05 10.54 16291600 10.54
25 2013-08-19 10.29 10.50 10.09 10.32 17854700 10.32
26 2013-08-16 10.80 10.90 10.40 10.51 29248600 10.51
27 2013-08-15 10.90 11.00 10.74 10.96 20780600 10.96
28 2013-08-14 11.18 11.56 10.77 11.04 44830500 11.04
29 2013-08-13 10.99 12.18 10.74 10.93 127211600 10.93
30 2013-08-12 10.65 10.93 10.12 10.78 92858900 10.78
31 2013-08-09 9.78 9.98 9.54 9.76 51295700 9.76
32 2013-08-08 9.23 9.40 9.15 9.23 8858400 9.23
33 2013-08-07 9.49 9.64 9.11 9.21 17327300 9.21
34 2013-08-06 9.64 9.88 9.41 9.57 24104900 9.57
35 2013-08-05 9.00 9.77 8.98 9.58 33620900 9.58
36 2013-08-02 8.77 8.94 8.72 8.92 8390200 8.92
37 2013-08-01 8.89 8.91 8.72 8.76 8673400 8.76
38 2013-07-31 8.65 8.91 8.65 8.84 13802700 8.84
39 2013-07-30 8.64 8.92 8.62 8.70 14920400 8.70
40 2013-07-29 8.80 8.90 8.57 8.67 17088500 8.67
41 2013-07-26 8.94 9.00 8.77 8.88 13484200 8.88
42 2013-07-25 8.95 9.08 8.90 8.98 11807800 8.98
43 2013-07-24 9.01 9.09 8.93 9.00 13026500 9.00
44 2013-07-23 9.14 9.18 8.95 9.02 13123600 9.02
45 2013-07-22 8.92 9.12 8.90 9.08 15415300 9.08
46 2013-07-19 9.24 9.28 8.90 8.97 23398400 8.97
47 2013-07-18 9.23 9.31 9.07 9.12 12500000 9.12
48 2013-07-17 9.07 9.32 9.07 9.24 15088100 9.24
49 2013-07-16 9.36 9.37 9.10 9.13 18645600 9.13
50 2013-07-15 9.20 9.55 9.18 9.37 17644700 9.37
51 2013-07-12 9.29 9.33 9.20 9.24 12874700 9.24
52 2013-07-11 9.28 9.35 9.16 9.33 17583600 9.33
53 2013-07-10 9.74 9.74 9.10 9.28 31079300 9.28
54 2013-07-09 9.61 9.93 9.55 9.64 29795600 9.64
55 2013-07-08 9.61 9.67 9.48 9.55 16762300 9.55
56 2013-07-05 9.65 9.88 9.50 9.55 19190100 9.55
57 2013-07-03 9.51 9.74 9.42 9.65 22608200 9.65
58 2013-07-02 10.20 10.27 9.63 9.70 44561500 9.70
59 2013-07-01 10.03 10.62 9.96 10.29 41076400 10.29
60 2013-06-28 10.71 10.98 10.25 10.46 136025600 10.46
61 2013-06-27 15.04 15.09 14.28 14.48 37407900 14.48
62 2013-06-26 14.70 15.00 14.70 14.91 23230800 14.91
63 2013-06-25 14.39 14.62 14.22 14.56 17641400 14.56
64 2013-06-24 13.50 14.29 13.50 14.10 19076300 14.10
65 2013-06-21 13.97 14.05 13.68 13.78 14323900 13.78
66 2013-06-20 13.92 14.18 13.82 13.98 16806700 13.98
67 2013-06-19 14.44 14.61 13.95 14.18 34716900 14.18
68 2013-06-18 14.32 15.00 14.26 14.84 31795000 14.84
69 2013-06-17 14.60 14.65 14.18 14.30 12316700 14.30
70 2013-06-14 14.64 14.93 14.35 14.44 24987700 14.44
71 2013-06-13 13.95 14.49 13.83 14.42 32443000 14.42
72 2013-06-12 13.67 13.79 13.50 13.56 9469200 13.56
73 2013-06-11 13.82 13.95 13.63 13.66 11907100 13.66
74 2013-06-10 13.87 14.09 13.70 13.95 16515300 13.95
75 2013-06-07 13.82 14.13 13.74 13.86 18496900 13.86
76 2013-06-06 13.56 13.90 13.53 13.71 15276600 13.71
77 2013-06-05 13.49 13.92 13.37 13.63 24596400 13.63
78 2013-06-04 13.57 13.90 13.48 13.54 15562300 13.54
79 2013-06-03 14.03 14.05 13.30 13.61 25458800 13.61
80 2013-05-31 14.50 14.52 13.83 13.96 30919100 13.96
81 2013-05-30 14.26 14.56 14.06 14.42 20257500 14.42
82 2013-05-29 14.30 14.54 14.16 14.17 17250300 14.17
83 2013-05-28 14.53 14.61 14.31 14.36 14657900 14.36
84 2013-05-24 14.46 14.65 14.36 14.48 13456300 14.48
85 2013-05-23 14.12 14.67 14.08 14.43 15343200 14.43
86 2013-05-22 14.60 14.84 14.34 14.46 20391200 14.46
87 2013-05-21 15.01 15.04 14.65 14.77 14376000 14.77
88 2013-05-20 14.70 15.27 14.53 14.98 18726400 14.98
89 2013-05-17 14.71 14.86 14.52 14.62 15557100 14.62
90 2013-05-16 15.07 15.13 14.60 14.69 20357600 14.69
91 2013-05-15 14.90 15.42 14.72 15.00 26858700 15.00
92 2013-05-14 16.01 16.14 14.92 15.25 52632100 15.25
93 2013-05-13 15.73 16.16 15.70 15.88 29220200 15.88
94 2013-05-10 15.49 15.68 15.31 15.54 17895600 15.54
95 2013-05-09 15.15 15.68 14.87 15.39 40657600 15.39
96 2013-05-08 15.10 15.15 14.80 14.90 17680100 14.90
97 2013-05-07 15.39 15.51 14.68 14.82 35314700 14.82
98 2013-05-06 15.66 15.98 15.45 15.57 23381500 15.57
99 2013-05-03 16.08 16.11 15.59 15.63 24469800 15.63
100 2013-05-02 15.65 16.06 15.30 15.70 26204800 15.70
101 2013-05-01 16.37 16.49 15.46 15.80 33440200 15.80
102 2013-04-30 15.79 16.59 15.75 16.29 42800400 16.29
103 2013-04-29 15.41 15.68 15.27 15.61 26672000 15.61
104 2013-04-26 14.84 15.09 14.66 15.02 17746700 15.02
105 2013-04-25 15.08 15.39 14.63 14.81 30676900 14.81
106 2013-04-24 14.48 14.96 14.38 14.90 24928000 14.90
107 2013-04-23 14.08 14.54 13.96 14.33 20400500 14.33
108 2013-04-22 13.85 14.13 13.61 13.99 19243400 13.99
109 2013-04-19 13.50 13.91 13.10 13.84 24013200 13.84
110 2013-04-18 13.72 14.04 13.38 13.50 26546600 13.50
111 2013-04-17 13.88 14.20 13.72 13.91 29646100 13.91
112 2013-04-16 13.97 14.15 13.85 13.92 19340500 13.92
113 2013-04-15 13.45 13.85 13.42 13.71 21250400 13.71
114 2013-04-12 13.67 14.04 13.38 13.64 34977000 13.64
115 2013-04-11 14.16 14.20 13.28 13.55 79542900 13.55
116 2013-04-10 14.86 14.99 14.47 14.69 36632400 14.69
117 2013-04-09 14.74 14.92 14.61 14.85 19527700 14.85
118 2013-04-08 14.89 15.06 14.61 14.67 27616900 14.67
119 2013-04-05 14.76 14.94 14.56 14.70 20961200 14.70
120 2013-04-04 15.09 15.31 14.92 14.99 27401500 14.99
121 2013-04-03 15.00 15.25 14.61 15.03 39206200 15.03
122 2013-04-02 15.20 15.47 15.04 15.14 31329600 15.14
123 2013-04-01 14.66 15.36 14.29 15.11 57405400 15.11
124 2013-03-28 15.18 15.55 14.14 14.45 107118000 14.45
125 2013-03-27 14.57 14.92 14.30 14.57 50966700 14.57
126 2013-03-26 14.57 14.73 14.25 14.46 36076800 14.46
127 2013-03-25 13.97 14.62 13.64 14.23 78015800 14.23
128 2013-03-22 16.43 16.82 14.64 14.91 123949500 14.91
129 2013-03-21 15.97 16.48 15.95 16.16 46990300 16.16
130 2013-03-20 15.95 16.44 15.67 16.00 84106500 16.00
131 2013-03-19 15.12 15.30 14.86 15.03 27680400 15.03
132 2013-03-18 14.79 15.59 14.65 15.02 51108500 15.02
133 2013-03-15 15.14 15.27 14.80 14.98 39219000 14.98
134 2013-03-14 15.82 16.01 15.04 15.06 96029000 15.06
135 2013-03-13 14.17 15.70 14.11 15.65 82044100 15.65
136 2013-03-12 15.22 15.39 14.04 14.47 84606100 14.47
137 2013-03-11 13.13 14.92 13.00 14.90 92815900 14.90
138 2013-03-08 13.53 13.53 13.02 13.06 21945800 13.06
139 2013-03-07 13.46 13.55 12.98 13.22 27168300 13.22
140 2013-03-06 12.66 13.56 12.56 13.35 49755000 13.35
141 2013-03-05 12.86 12.92 12.55 12.58 27086200 12.58
142 2013-03-04 13.20 13.28 12.76 12.81 26847900 12.81
143 2013-03-01 13.44 13.78 13.03 13.26 39291800 13.26
144 2013-02-28 13.15 13.79 12.94 13.36 43004000 13.36
145 2013-02-27 12.93 13.68 12.75 13.25 38537000 13.25
146 2013-02-26 13.34 13.45 12.56 13.06 51689800 13.06
147 2013-02-25 13.66 13.76 13.05 13.25 35725200 13.25
148 2013-02-22 13.81 13.98 13.04 13.18 42572100 13.18
149 2013-02-21 13.94 14.05 13.35 13.88 49801500 13.88
150 2013-02-20 13.99 14.38 13.65 13.71 55153100 13.71
151 2013-02-19 14.25 14.82 14.17 14.36 46121000 14.36
152 2013-02-15 15.30 15.49 13.96 14.16 92805500 14.16
153 2013-02-14 13.20 15.16 12.94 15.07 112596700 15.07
154 2013-02-13 15.10 15.39 13.81 13.99 77515100 13.99
155 2013-02-12 15.47 16.07 15.15 15.20 44203400 15.20
156 2013-02-11 15.72 16.18 15.29 15.73 60717500 15.73
157 2013-02-08 17.15 17.22 16.44 16.49 61324600 16.49
158 2013-02-07 16.68 16.98 16.20 16.96 75845700 16.96
159 2013-02-06 15.52 16.54 15.28 16.05 100512100 16.05
160 2013-02-05 15.79 16.89 15.53 16.02 127576800 16.02
161 2013-02-04 13.71 15.01 13.52 14.98 82670900 14.98
162 2013-02-01 13.45 13.55 12.15 13.03 59354800 13.03
163 2013-01-31 12.45 13.35 12.15 12.98 111855800 12.98
164 2013-01-30 16.08 16.62 13.72 13.78 224097000 13.78
165 2013-01-29 15.92 16.00 14.68 15.66 106906200 15.66
166 2013-01-28 17.82 17.89 16.03 16.18 88895700 16.18
167 2013-01-25 18.09 18.24 17.40 17.54 57585500 17.54
168 2013-01-24 16.58 18.32 16.33 17.74 107804000 17.74
169 2013-01-23 17.87 18.08 17.16 17.35 75614300 17.35
170 2013-01-22 17.07 17.90 16.90 17.90 110834000 17.90
171 2013-01-18 15.86 15.94 15.34 15.84 71198300 15.84
172 2013-01-17 14.89 14.92 14.46 14.91 38902900 14.91
173 2013-01-16 14.75 15.36 14.65 14.74 67643400 14.74
174 2013-01-15 14.90 15.06 14.24 14.48 59086400 14.48
175 2013-01-14 13.95 15.09 13.90 14.95 143607300 14.95
176 2013-01-11 11.73 13.60 11.68 13.56 111026800 13.56
177 2013-01-10 11.85 11.98 11.61 11.92 24542500 11.92
178 2013-01-09 11.94 12.02 11.46 11.48 23609900 11.48
179 2013-01-08 11.99 12.24 11.74 11.91 33655500 11.91
180 2013-01-07 12.08 12.14 11.72 11.95 21292900 11.95
181 2013-01-04 11.64 12.03 11.44 11.95 30667000 11.95
182 2013-01-03 11.80 11.84 11.40 11.47 25866100 11.47
183 2013-01-02 12.32 12.37 11.64 11.72 29400600 11.72
184 2012-12-31 11.83 11.97 11.67 11.87 25606900 11.87
185 2012-12-28 11.70 12.10 11.57 11.79 28917100 11.79
186 2012-12-27 12.06 12.16 11.50 11.76 51026900 11.76
187 2012-12-26 10.79 12.03 10.66 11.83 70807700 11.83
188 2012-12-24 10.72 10.98 10.59 10.61 28912100 10.61
189 2012-12-21 11.84 12.39 10.91 10.91 149196000 10.91
190 2012-12-20 13.64 14.13 13.53 14.12 49596800 14.12
191 2012-12-19 13.55 13.89 13.52 13.63 26795900 13.63
192 2012-12-18 13.75 13.88 13.28 13.72 39727800 13.72
193 2012-12-17 14.15 14.20 13.66 13.93 44063800 13.93
194 2012-12-14 13.86 14.21 13.53 14.04 53970900 14.04
195 2012-12-13 13.45 14.16 13.42 13.86 82171300 13.86
196 2012-12-12 12.61 13.34 12.36 13.31 63731100 13.31
197 2012-12-11 11.96 12.67 11.88 12.60 43231600 12.60
198 2012-12-10 11.98 12.12 11.66 11.88 24329200 11.88
199 2012-12-07 11.85 12.13 11.75 12.01 24527000 12.01
200 2012-12-06 12.01 12.18 11.62 11.89 33156700 11.89
201 2012-12-05 11.62 12.12 11.47 11.94 32036200 11.94
202 2012-12-04 11.59 11.73 11.44 11.56 21189500 11.56
203 2012-12-03 11.39 11.82 11.25 11.60 38289200 11.60
204 2012-11-30 11.65 11.84 11.38 11.60 35486800 11.60
205 2012-11-29 12.28 12.30 11.37 11.54 85005500 11.54
206 2012-11-28 10.31 11.26 10.20 11.10 53958900 11.10
207 2012-11-27 11.98 12.00 10.71 10.72 84235200 10.72
208 2012-11-26 11.96 12.14 11.44 11.98 87527400 11.98
209 2012-11-23 11.60 11.86 11.26 11.66 74491400 11.66
210 2012-11-21 9.66 10.35 9.62 10.26 39520700 10.26
211 2012-11-20 10.02 10.04 9.57 9.71 49958400 9.71
212 2012-11-19 9.15 9.80 9.03 9.59 42524400 9.59
213 2012-11-16 8.85 9.30 8.53 9.20 35216100 9.20
214 2012-11-15 8.47 8.84 8.47 8.79 19957600 8.79
215 2012-11-14 8.52 8.67 8.38 8.49 15260800 8.49
216 2012-11-13 8.80 8.85 8.31 8.40 22877800 8.40
217 2012-11-12 9.05 9.07 8.67 8.81 20674900 8.81
218 2012-11-09 8.20 8.70 8.20 8.54 20708700 8.54
219 2012-11-08 8.46 8.52 8.14 8.20 16511200 8.20
220 2012-11-07 9.12 9.15 8.23 8.24 35905800 8.24
221 2012-11-06 9.01 9.08 8.87 9.06 18275700 9.06
222 2012-11-05 8.80 9.17 8.74 8.90 28343600 8.90
223 2012-11-02 8.84 8.91 8.69 8.71 24117600 8.71
224 2012-11-01 8.06 8.85 8.01 8.70 39524200 8.70
225 2012-10-31 7.59 8.00 7.53 7.93 13113900 7.93
226 2012-10-26 7.70 7.79 7.51 7.57 11489100 7.57
227 2012-10-25 7.64 7.77 7.53 7.71 10854300 7.71
228 2012-10-24 7.68 7.69 7.50 7.52 9163800 7.52
229 2012-10-23 7.59 7.90 7.46 7.62 15294900 7.62
230 2012-10-22 7.78 7.84 7.56 7.69 11932900 7.69
231 2012-10-19 7.88 7.90 7.64 7.76 19095100 7.76
232 2012-10-18 7.99 8.07 7.83 7.84 16355600 7.84
233 2012-10-17 7.80 8.08 7.78 7.99 14365900 7.99
234 2012-10-16 7.82 7.88 7.72 7.83 11589300 7.83
235 2012-10-15 7.83 7.90 7.71 7.80 9162500 7.80
236 2012-10-12 7.88 8.03 7.72 7.80 13505600 7.80
237 2012-10-11 7.82 8.00 7.77 7.91 11652000 7.91
238 2012-10-10 7.76 7.86 7.58 7.76 19016500 7.76
239 2012-10-09 8.06 8.11 7.77 7.80 23247400 7.80
240 2012-10-08 8.22 8.45 8.10 8.24 12219700 8.24
241 2012-10-05 8.35 8.44 8.18 8.22 15775700 8.22
242 2012-10-04 8.13 8.27 8.04 8.23 19958600 8.23
243 2012-10-03 8.35 8.35 8.00 8.06 27573900 8.06
244 2012-10-02 7.90 8.44 7.83 8.27 55091100 8.27
245 2012-10-01 7.62 7.88 7.27 7.86 52534300 7.86
246 2012-09-28 8.20 8.20 7.46 7.50 139438500 7.50
247 2012-09-27 7.20 7.23 6.75 7.14 71975800 7.14
248 2012-09-26 6.84 7.09 6.63 7.00 43862900 7.00
249 2012-09-25 6.37 6.71 6.25 6.60 45954900 6.60
250 2012-09-24 6.37 6.45 6.22 6.31 20380000 6.31
251 2012-09-21 6.80 6.84 6.43 6.46 47179100 6.46
252 2012-09-20 7.11 7.22 6.90 6.91 14097400 6.91
253 2012-09-19 7.48 7.48 7.18 7.21 15151800 7.21
254 2012-09-18 7.17 7.81 7.15 7.42 35720600 7.42
255 2012-09-17 7.39 7.46 7.18 7.25 14519100 7.25
256 2012-09-14 7.44 7.86 7.43 7.56 21556300 7.56
257 2012-09-13 7.38 7.59 7.30 7.43 14791300 7.43
258 2012-09-12 7.54 7.60 7.24 7.42 14702500 7.42
259 2012-09-11 7.18 7.55 7.12 7.46 17342600 7.46
260 2012-09-10 7.14 7.25 7.00 7.15 14868100 7.15
261 2012-09-07 6.73 7.22 6.67 7.19 23231000 7.19
262 2012-09-06 6.59 6.80 6.33 6.72 22312100 6.72
263 2012-09-05 6.61 6.75 6.38 6.52 7858000 6.52
264 2012-09-04 6.66 6.69 6.57 6.65 9693500 6.65

BIN
by-sa.pdf Normal file

Binary file not shown.

BIN
by.pdf Normal file

Binary file not shown.

37
cc.tex Normal file
View file

@ -0,0 +1,37 @@
\includegraphics[height=7mm,keepaspectratio=true]{by-sa}\\%
Cette création est mise à disposition selon le contrat
\href{http://creativecommons.org/licenses/by-sa/2.5/ca/deed.fr}{%
Paternité-Partage à l'identique 2.5 Canada} de Creative Commons
disponible à l'adresse \\
http://creativecommons.org/licenses/by-sa/2.5/ca/deed.fr \\
En vertu de ce contrat, vous êtes libre de :
\begin{itemize}
\item \textbf{partager} --- reproduire, distribuer et communiquer
l'{\oe}uvre;
\item \textbf{remixer} --- adapter l'{\oe}uvre;
\item utiliser cette {\oe}uvre à des fins commerciales.
\end{itemize}
Selon les conditions suivantes:\\
\begin{tabularx}{\linewidth}{@{}lX@{}}
\raisebox{-9mm}[0mm][13mm]{%
\includegraphics[height=11mm,keepaspectratio=true]{by}} &
\textbf{Attribution} --- Vous devez attribuer l'{\oe}uvre de la
manière indiquée par l'auteur de l'{\oe}uvre ou le titulaire des
droits (mais pas d'une manière qui suggérerait qu'ils vous
soutiennent ou
approuvent votre utilisation de l'{\oe}uvre). \\
\raisebox{-9mm}{\includegraphics[height=11mm,keepaspectratio=true]{sa}}
& \textbf{Partage à l'identique} --- Si vous modifiez, transformez
ou adaptez cette {\oe}uvre, vous n'avez le droit de distribuer
votre création que sous une licence identique ou similaire à
celle-ci.
\end{tabularx}
%%% Local Variables:
%%% mode: latex
%%% TeX-master: t
%%% End:

62
cg130823a001-fra.csv Normal file
View file

@ -0,0 +1,62 @@
Date Variation
08-07 3.4
08-08 3.5
08-09 3.4
08-10 2.6
08-11 2.0
08-12 1.2
09-01 1.1
09-02 1.4
09-03 1.2
09-04 0.4
09-05 0.1
09-06 -0.3
09-07 -0.9
09-08 -0.8
09-09 -0.9
09-10 0.1
09-11 1.0
09-12 1.3
10-01 1.9
10-02 1.6
10-03 1.4
10-04 1.8
10-05 1.4
10-06 1.0
10-07 1.8
10-08 1.7
10-09 1.9
10-10 2.4
10-11 2.0
10-12 2.4
11-01 2.3
11-02 2.2
11-03 3.3
11-04 3.3
11-05 3.7
11-06 3.1
11-07 2.7
11-08 3.1
11-09 3.2
11-10 2.9
11-11 2.9
11-12 2.3
12-01 2.5
12-02 2.6
12-03 1.9
12-04 2.0
12-05 1.2
12-06 1.5
12-07 1.3
12-08 1.2
12-09 1.2
12-10 1.2
12-11 0.8
12-12 0.8
13-01 0.5
13-02 1.2
13-03 1.0
13-04 0.4
13-05 0.7
13-06 1.2
13-07 1.3
1 Date Variation
2 08-07 3.4
3 08-08 3.5
4 08-09 3.4
5 08-10 2.6
6 08-11 2.0
7 08-12 1.2
8 09-01 1.1
9 09-02 1.4
10 09-03 1.2
11 09-04 0.4
12 09-05 0.1
13 09-06 -0.3
14 09-07 -0.9
15 09-08 -0.8
16 09-09 -0.9
17 09-10 0.1
18 09-11 1.0
19 09-12 1.3
20 10-01 1.9
21 10-02 1.6
22 10-03 1.4
23 10-04 1.8
24 10-05 1.4
25 10-06 1.0
26 10-07 1.8
27 10-08 1.7
28 10-09 1.9
29 10-10 2.4
30 10-11 2.0
31 10-12 2.4
32 11-01 2.3
33 11-02 2.2
34 11-03 3.3
35 11-04 3.3
36 11-05 3.7
37 11-06 3.1
38 11-07 2.7
39 11-08 3.1
40 11-09 3.2
41 11-10 2.9
42 11-11 2.9
43 11-12 2.3
44 12-01 2.5
45 12-02 2.6
46 12-03 1.9
47 12-04 2.0
48 12-05 1.2
49 12-06 1.5
50 12-07 1.3
51 12-08 1.2
52 12-09 1.2
53 12-10 1.2
54 12-11 0.8
55 12-12 0.8
56 13-01 0.5
57 13-02 1.2
58 13-03 1.0
59 13-04 0.4
60 13-05 0.7
61 13-06 1.2
62 13-07 1.3

BIN
constructionserieAR.ods Normal file

Binary file not shown.

13
ex2.9.wxm Normal file
View file

@ -0,0 +1,13 @@
/* [wxMaxima batch file version 1] [ DO NOT EDIT BY HAND! ]*/
/* [ Created with wxMaxima version 13.04.2 ] */
/* [wxMaxima: input start ] */
algsys([(1+t^2)*E - (1+a^2)*V - D = 0, t*E = a*V], [t,E]);
/* [wxMaxima: input end ] */
/* [wxMaxima: input start ] */
float(algsys([(1+t^2)*E - (1+0.5^2)*0.04 - 0.01 = 0, t*E = 0.5*0.04], [t,E]));
/* [wxMaxima: input end ] */
/* Maxima can't load/batch files which end with a comment! */
"Created with wxMaxima"$

145
exercices_series_chrono.tex Normal file
View file

@ -0,0 +1,145 @@
\documentclass[letterpaper,10pt]{memoir}
\usepackage[T1]{fontenc}
\usepackage[utf8]{inputenc}
\usepackage[english,francais]{babel}
\usepackage[autolanguage]{numprint}
\usepackage{vgmath,vgsets,amsmath,icomma}
\usepackage{pslatex}
\usepackage[sc]{mathpazo}
\usepackage[noae]{Sweave}
\usepackage{graphicx,color}
\usepackage{longtable,lscape} % table de loi F
\usepackage[absolute]{textpos}
\usepackage{answers}
\usepackage[alwaysadjust,defblank]{paralist}
%%% Hyperliens
\usepackage{hyperref}
\definecolor{link}{rgb}{0,0,0.3}
\hypersetup{
pdftex,
colorlinks,%
citecolor=link,%
filecolor=link,%
linkcolor=link,%
urlcolor=link}
%%% Page titre
\title{\HUGE
\fontseries{ub}\selectfont Séries chronologiques \\[0.5\baselineskip]
\huge\fontseries{m}\selectfont Exercices et solutions}
\author{\LARGE François Pelletier \\[3mm]
\large École d'actuariat \\ Université Laval \\[6mm]
}
\date{Première édition - Automne 2013}
\newcommand{\ISBN}{}
%%% Sous-figures
\newsubfloat{figure}
%%% Style des entêtes de chapitres
\chapterstyle{hangnum}
%%% Styles des entêtes et pieds de page
\setlength{\marginparsep}{7mm}
\setlength{\marginparwidth}{13mm}
\setlength{\headwidth}{\textwidth}
\addtolength{\headwidth}{\marginparsep}
\addtolength{\headwidth}{\marginparwidth}
%%% Style de la bibliographie
\bibliographystyle{francais}
%%% Options de babel
\frenchbsetup{CompactItemize=false,%
ThinSpaceInFrenchNumbers=true}
\addto\captionsfrench{\def\tablename{{\scshape Tab.}}}
\addto\captionsfrench{\def\figurename{{\scshape Fig.}}}
%%% Associations entre les environnements et les fichiers
\Newassociation{sol}{solution}{solutions}
\Newassociation{rep}{reponse}{reponses}
%%% Environnement pour les exercices
\newcounter{exercice}[chapter]
\newenvironment{exercice}{%
\begin{list}{\bfseries \arabic{chapter}.\arabic{exercice}}{%
\refstepcounter{exercice}
\settowidth{\labelwidth}{\bfseries \arabic{chapter}.\arabic{exercice}}
\setlength{\leftmargin}{\labelwidth}
\addtolength{\leftmargin}{\labelsep}
\setdefaultenum{a)}{i)}{}{}}\item}
{\end{list}}
%%% Environnement pour les réponses
\renewenvironment{reponse}[1]{%
\begin{list}{\bfseries #1}{%
\settowidth{\labelwidth}{#1}
\setlength{\leftmargin}{\labelwidth}
\addtolength{\leftmargin}{\labelsep}
\setdefaultenum{a)}{i)}{}{}}\item}
{\end{list}}
\renewcommand{\reponseparams}{{\thechapter.\theexercice}}
%%% Environnement pour les solutions
\renewenvironment{solution}[1]{%
\begin{list}{\bfseries #1}{%
\settowidth{\labelwidth}{#1}
\setlength{\leftmargin}{\labelwidth}
\addtolength{\leftmargin}{\labelsep}
\setdefaultenum{a)}{i)}{}{}}\item}
{\end{list}}
\renewcommand{\solutionparams}{{\thechapter.\theexercice}}
%%% Nouvelles commandes
\newcommand{\cov}[1]{\mathrm{Cov} ( #1 )}
\renewcommand{\Cov}[1]{\mathrm{Cov}\! \left( #1 \right)}
\newcommand{\prob}[1]{\mathrm{Pr} [ #1 ]}
\newcommand{\Prob}[1]{\mathrm{Pr}\! \left[ #1 \right]}
\newcommand{\MSE}{\mathrm{MSE}}
%%% Environnements pour le code S: police plus petite
\RecustomVerbatimEnvironment{Sinput}{Verbatim}{fontshape=sl,fontsize=\small}
\RecustomVerbatimEnvironment{Soutput}{Verbatim}{fontsize=\small}
\RecustomVerbatimEnvironment{Scode}{Verbatim}{fontsize=\small}
%%% Un petit peu d'aide pour la césure
\hyphenation{con-fiance}
\hyphenation{con-train-te}
\DeclareMathOperator{\sgn}{sgn}
\begin{document}
\frontmatter
\pagestyle{empty}
\include{pagetitre}
\pagestyle{companion}
\cleardoublepage
\tableofcontents
\mainmatter
\include{serie1-exercices}
\include{serie2-exercices}
\include{serie3-exercices}
\appendix
\include{solutions}
\pagestyle{empty}
\include{cc}
\cleardoublepage
\cleartoverso
\pagestyle{empty}
\bandeverso
\end{document}
%%% Local Variables:
%%% mode: latex
%%% TeX-master: t
%%% End:

62
inflation.csv Normal file
View file

@ -0,0 +1,62 @@
Date Variation
08-07 -0.0945362195233141
08-08 0.47143001707356
08-09 0.694899473895187
08-10 0.926830036087345
08-11 1.40493767270043
08-12 2.33578782869081
09-01 1.51881892761456
09-02 0.893102534273033
09-03 2.15125064804327
09-04 0.833569439522224
09-05 0.151752726872046
09-06 0.293669604936261
09-07 0.965236773198128
09-08 0.261635333843741
09-09 0.793085952121019
09-10 0.392985995257477
09-11 1.64858925157916
09-12 2.00885858394999
10-01 3.2071243267656
10-02 2.28400647411436
10-03 1.39035409590138
10-04 0.645193173269466
10-05 0.73967588204237
10-06 1.07123858251644
10-07 -0.187932226942878
10-08 1.41644197839735
10-09 0.933495217399702
10-10 1.44524882331813
10-11 1.40195117395518
10-12 1.87431941830127
11-01 3.096886710615
11-02 2.13009020709946
11-03 2.74117598615254
11-04 1.67980638043298
11-05 1.72072154180839
11-06 0.121483912242173
11-07 0.883390358370279
11-08 1.58112311535382
11-09 1.59299597087813
11-10 2.52002400968706
11-11 2.37033548923
11-12 2.63367999928504
12-01 1.96211412213629
12-02 3.2046353136476
12-03 2.88554778615085
12-04 1.41273954079782
12-05 1.13704465501818
12-06 1.26921435539248
12-07 1.43036564553818
12-08 1.41762904317422
12-09 1.47470469386241
12-10 1.68213582395297
12-11 2.29324245139441
12-12 2.44852263639074
13-01 2.98231393269508
13-02 2.29727985225593
13-03 2.2792491554878
13-04 1.94771402102956
13-05 1.68656618714032
13-06 0.494577142535042
13-07 0.88677947797925
1 Date Variation
2 08-07 -0.0945362195233141
3 08-08 0.47143001707356
4 08-09 0.694899473895187
5 08-10 0.926830036087345
6 08-11 1.40493767270043
7 08-12 2.33578782869081
8 09-01 1.51881892761456
9 09-02 0.893102534273033
10 09-03 2.15125064804327
11 09-04 0.833569439522224
12 09-05 0.151752726872046
13 09-06 0.293669604936261
14 09-07 0.965236773198128
15 09-08 0.261635333843741
16 09-09 0.793085952121019
17 09-10 0.392985995257477
18 09-11 1.64858925157916
19 09-12 2.00885858394999
20 10-01 3.2071243267656
21 10-02 2.28400647411436
22 10-03 1.39035409590138
23 10-04 0.645193173269466
24 10-05 0.73967588204237
25 10-06 1.07123858251644
26 10-07 -0.187932226942878
27 10-08 1.41644197839735
28 10-09 0.933495217399702
29 10-10 1.44524882331813
30 10-11 1.40195117395518
31 10-12 1.87431941830127
32 11-01 3.096886710615
33 11-02 2.13009020709946
34 11-03 2.74117598615254
35 11-04 1.67980638043298
36 11-05 1.72072154180839
37 11-06 0.121483912242173
38 11-07 0.883390358370279
39 11-08 1.58112311535382
40 11-09 1.59299597087813
41 11-10 2.52002400968706
42 11-11 2.37033548923
43 11-12 2.63367999928504
44 12-01 1.96211412213629
45 12-02 3.2046353136476
46 12-03 2.88554778615085
47 12-04 1.41273954079782
48 12-05 1.13704465501818
49 12-06 1.26921435539248
50 12-07 1.43036564553818
51 12-08 1.41762904317422
52 12-09 1.47470469386241
53 12-10 1.68213582395297
54 12-11 2.29324245139441
55 12-12 2.44852263639074
56 13-01 2.98231393269508
57 13-02 2.29727985225593
58 13-03 2.2792491554878
59 13-04 1.94771402102956
60 13-05 1.68656618714032
61 13-06 0.494577142535042
62 13-07 0.88677947797925

108
pagetitre.tex Normal file
View file

@ -0,0 +1,108 @@
%%% Police de caractère pour la page titre
\renewcommand{\sfdefault}{hls}
%%% Marge de gauche (1/3 de la page)
\newlength{\gauche}
\addtolength{\gauche}{72mm}
\addtolength{\gauche}{-\spinemargin}
%%% Épaisseur de la bande sur la page titre
\newlength{\ruleheight}
\setlength{\ruleheight}{7.75mm}
%%% Définition de la bande
\definecolor{gray}{gray}{0.4}
\textblockorigin{0mm}{279mm}
\newcommand{\banderecto}{%
\begin{textblock*}{71.5mm}[0,1](0mm,-46.5mm)
\textblockcolor{black} \rule{0mm}{\ruleheight}
\end{textblock*}
\begin{textblock*}{144mm}[0,1](72mm,-46.5mm)
\textblockcolor{gray} \rule{0mm}{\ruleheight}
\end{textblock*}}
\newcommand{\bandeverso}{%
\begin{textblock*}{144mm}[0,1](0mm,-46.5mm)
\textblockcolor{gray} \rule{0mm}{\ruleheight}
\end{textblock*}
\begin{textblock*}{71.5mm}[0,1](144.5mm,-46.5mm)
\textblockcolor{black} \rule{0mm}{\ruleheight}
\end{textblock*}}
%%% Titre
\begin{adjustwidth*}{\gauche}{-15mm}
\sffamily\fontseries{ub}\selectfont
\raggedright
\vspace*{2cm}
\thetitle
\end{adjustwidth*}
%%% Affichage de la bande
\banderecto
\cleardoublepage
%%% Page de garde
\begin{adjustwidth*}{\gauche}{-15mm}
\sffamily\fontseries{ub}\selectfont
\raggedright
\vspace*{2cm}
\thetitle \\
\bfseries
\vspace*{3cm}
\theauthor
\vspace*{\fill}
\thedate
\end{adjustwidth*}
\clearpage
%%% Page de notices
\begingroup
\calccentering{\unitlength}
\begin{adjustwidth*}{\unitlength}{-\unitlength}
\small
\setlength{\parindent}{0pt}
\setlength{\parskip}{\baselineskip}
{\textcopyright} 2013 François Pelletier \\
\includegraphics[height=7mm,keepaspectratio=true]{by}\;%
\includegraphics[height=7mm,keepaspectratio=true]{sa} \\
Cette création est mise à disposition selon le contrat
Paternité-Partage des conditions initiales à l'identique 2.5 Canada
disponible en ligne
\url{http://creativecommons.org/licenses/by-sa/2.5/ca/} ou par
courrier postal à Creative Commons, 171 Second Street, Suite 300,
San Francisco, California 94105, USA.
\textbf{Historique de publication}
\vspace{-\baselineskip}
\begin{tabbing}
Décembre 2013:\quad\= Première édition
\end{tabbing}
\textbf{Code source} \\
Le code source {\LaTeX} de ce document est disponible à l'adresse
\begin{quote}
\url{https://github.com/franc00018/ACT-2010-Exercices}
\end{quote}
ou en communiquant directement avec les auteurs.
\vspace{1cm}
% ISBN \ISBN \\
% Dépôt légal -- Bibliothèque et Archives nationales du Québec, 2014 \\
% Dépôt légal -- Bibliothèque et Archives Canada, 2014
\end{adjustwidth*}
\endgroup
%%% Retour à la police normale
\renewcommand{\sfdefault}{phv}
\clearpage
%%% Local Variables:
%%% mode: latex
%%% TeX-master: "exercices_series_chrono"
%%% coding: utf-8-unix
%%% End:

7
processusar2-estim.r Normal file
View file

@ -0,0 +1,7 @@
set.seed(123)
(serie <- arima.sim(n = 10, list(ar = c(0.5,-0.25))))
acfserie <- acf(serie,type="correlation")$acf[c(2,3)]
acfserie[1]*(1-acfserie[2]) / (1-acfserie[1]^2)
(acfserie[2] - acfserie[1]^2) / (1-acfserie[1]^2)

View file

@ -0,0 +1,3 @@
\bigskip
\section*{Réponses}

View file

@ -0,0 +1,3 @@
\bigskip
\section*{Réponses}

View file

@ -0,0 +1,3 @@
\bigskip
\section*{Réponses}

BIN
sa.pdf Normal file

Binary file not shown.

536
serie1-exercices.Rnw Normal file
View file

@ -0,0 +1,536 @@
\chapter{Méthodes de lissage et saisonnalité}
\label{chap:methodes-lissage}
\Opensolutionfile{reponses}[reponses-methodes-lissage]
\Opensolutionfile{solutions}[solutions-methodes-lissage]
\begin{Filesave}{reponses}
\bigskip
\section*{Réponses}
\end{Filesave}
\begin{Filesave}{solutions}
\section*{Chapitre \ref{chap:methodes-lissage}}
\addcontentsline{toc}{section}{Chapitre \protect\ref{chap:methodes-lissage}}
\end{Filesave}
<<echo=FALSE>>=
options(width = 55)
@
\begin{exercice}
On considère les taux d'inflation sur 12 mois disponibles dans le
fichier \url{cg130823a001-fra.csv}. On représente cette série chronologique
par la variable aléatoire $Y_t$. L'an passé, les Canadiens ont dépensé
en moyenne $674\$$ en cadeaux au mois de décembre 2012. Notre objectif
est de prévoir quel sera le montant dépensé pour l'achat de cadeaux en
décembre 2013.
\begin{enumerate}
\item Tracez un graphique de la série chronologique $Y_t$ à l'aide
d'un logiciel statistique. Êtes vous en mesure de déceler
visuellement la présence d'une tendance et/ou d'une saisonnalité ?
\item Utilisez l'opérateur différentiel $\nabla_{12}$ afin d'éliminer
la saisonnalité annuelle de la série chronologique $Y_t$ et obtenir
la série $Z_t$. Tracez à nouveau un graphique avec les données
obtenues. Remarquez-vous toujours la présence de saisonnalité ?
Tracez le graphique de la composante de saisonnalité $s_t$.
\item Maintenant, nous voulons déceler s'il y a présence d'une
tendance dans les données. En utilisant la méthode de la moyenne
mobile avec $q=1$ et $q=5$, du lissage exponentiel double avec
$\alpha=5\%$ et de la régression linéaire simple, estimer la
tendance $\hat{m}_t$. Faire le graphique superposé des 5 tendances.
\item En utilisant le résultat de la régression linéaire précédente,
prévoir la valeur non saisonnalisée en décembre 2013. En évaluant la
moyenne des différences entre la série $Y_t$ et la valeur de la
régression pour les mois de décembre des années précédente, on peut
estimer la valeur $\hat{s}_{12}$. Ajouter cette valeur au résultat
obtenu pour obtenir une estimation du taux d'inflation en décembre
2013.
\item En applicant ce taux d'inflation à la donnée du problème,
prédire le montant dépensé pour l'achat de cadeaux en décembre 2013.
\end{enumerate}
\begin{sol}
\begin{enumerate}
\item
<<>>=
library(xtable)
library(zoo)
library(TTR)
Yt <- read.csv("inflation.csv",header=TRUE,sep="\t")[,2]
Yt.ts <-ts(Yt,start=c(2008,7),deltat=1/12)
@
<<results=tex>>=
xtable(Yt.ts,digits=1) ## Générer une table LaTeX
@
<<echo=FALSE>>=
pdf("exercice1-graph1.pdf",paper="special", width=6,
height=6) ## Générer le fichier PDF contenant le graphique
plot(Yt.ts)
dummy <- dev.off() ## Fermer le fichier PDF
@
On retrouve le graphique de la série $Y_t$ à la figure \ref{fig:exercice1-graph1}.
\begin{figure}[!ht]
\centering
\includegraphics[height=4in, width=4in]{exercice1-graph1.pdf}
\caption{Graphique de la série $Y_t$}
\label{fig:exercice1-graph1}
\end{figure}
\item
<<results=tex>>=
xtable(Zt.ts <- diff(Yt.ts,12),digits=1)
@
<<echo=FALSE>>=
pdf("exercice1-graph2.pdf",paper="special", width=6,
height=6)
plot(Zt.ts)
dummy <- dev.off()
@
On retrouve le graphique de la série désaisonnalisée $Z_t$ à la figure \ref{fig:exercice1-graph2}.
\begin{figure}[!ht]
\centering
\includegraphics[height=4in, width=4in]{exercice1-graph2.pdf}
\caption{Graphique de la série désaisonnalisée $Z_t$}
\label{fig:exercice1-graph2}
\end{figure}
On élimine la composante de saisonnalité
<<results=tex>>=
xtable(Yt.ts-Zt.ts,digits=1)
@
<<echo=FALSE>>=
pdf("exercice1-graph3.pdf",paper="special", width=6,
height=6)
plot(Zt.ts-Yt.ts)
dummy <- dev.off()
@
On retrouve le graphique de la composante de saisonnalité $Y_t-Z_t$ à la figure \ref{fig:exercice1-graph3}.
\begin{figure}[!ht]
\centering
\includegraphics[height=4in, width=4in]{exercice1-graph3.pdf}
\caption{Graphique de la composante de saisonnalité $Y_t-Z_t$}
\label{fig:exercice1-graph3}
\end{figure}
\item
On élimime maintenant la tendance:
On utilise une moyenne mobile avec $q=1$. Comme la fonction \emph{SMA()} utilise les $2q+1$ données précédentes et que nous voulons une moyenne mobile centrée, nous devons utiliser l'opérateur de rétrodécalage $B()$ pour décaler la série.
<<results=tex>>=
## Simple Moving Average(q=1)
xtable(mt1 <- lag(SMA(Zt.ts,n=3),1),digits=2)
@
Moyenne mobile avec $q=5$
<<results=tex>>=
## Simple Moving Average(q=5)
xtable(mt2 <- lag(SMA(Zt.ts,n=11),5),digits=2)
@
Lissage exponentiel double avec $\alpha=0.75$
<<results=tex>>=
## Double Exponential Moving Average
xtable(mt3 <- DEMA(Zt.ts,n=1,ratio=.05),digits=2)
@
Régression linéaire
<<>>=
t <- 0:48
(lm1 <- lm(Zt.ts~t)) ## Modèle de régression sur une variable
coeff1 <- coefficients(lm1)
@
<<results=tex>>=
xtable(mt4 <- ts(coeff1[1]+t*coeff1[2],start=c(2009,7),deltat=1/12),digits=2)
@
<<echo=FALSE>>=
pdf("exercice1-graph4.pdf",paper="special", width=6,
height=6)
plot(mt1,type="l",lty=1)
lines(mt2,type="l",lty=2)
lines(mt3,type="l",lty=3)
lines(mt4,type="l",lty=4)
dummy <- dev.off()
@
On retrouve le graphique de la tendance $m_t$ à la figure \ref{fig:exercice1-graph4}.
\begin{figure}[!ht]
\centering
\includegraphics[height=4in, width=4in]{exercice1-graph4.pdf}
\caption{Graphique de la tendance $m_t$}
\label{fig:exercice1-graph4}
\end{figure}
\item
<<>>=
projection <- coeff1[1]+53*coeff1[2]
saisonnalite <- mean((Yt.ts-Zt.ts)[6+12*0:3])
(taux.inf.dec.2013 <- (projection+saisonnalite))
@
Le taux d'inflation prejeté en décembre 2013 est \Sexpr{round(taux.inf.dec.2013,2)}\%
\item
<<>>=
depense.dec.2008 <- 674
depense.dec.2013 <- 674*(1+taux.inf.dec.2013/100)
@
Le montant projeté des achats de cadeaux en décembre 2013 est \Sexpr{round(depense.dec.2013,2)} \$
\end{enumerate}
\end{sol}
\end{exercice}
\begin{exercice}
On a estimé la tendance dun ensemble de données dincendie pour une
année. Cependant, suite à un problème informatique, certaines données
sont manquantes. Identifiez ces données.\\
\begin{tabular}{|l|l|l|l|}
\hline
\multicolumn{1}{|l|}{Mois} & \multicolumn{1}{l|}{Incendies} & \multicolumn{1}{l|}{Moyenne Mobile} & \\ \hline
1 & 4 & \multicolumn{1}{l|}{-} & \\ \hline
2 & 3 & \multicolumn{1}{l|}{-} & \\ \hline
3 & \multicolumn{1}{l|}{a} & 4,8 & \\ \hline
4 & \multicolumn{1}{l|}{b} & 4,8 & \\ \hline
5 & 2 & 5,4 & \\ \hline
6 & 4 & 5,2 & \\ \hline
7 & 6 & 3,6 & \\ \hline
8 & \multicolumn{1}{l|}{c} & 3,6 & \\ \hline
9 & \multicolumn{1}{l|}{0} & 4,4 & \\ \hline
10 & 2 & 3,8 & \\ \hline
11 & 8 & \multicolumn{1}{l|}{-} & \\ \hline
12 & 3 & \multicolumn{1}{l|}{-} & \\ \hline
\end{tabular}
\begin{sol}
On remarque d'abord que $q=2$.
On peut ensuite poser les équations suivantes:
\begin{align}
\label{eq:1}
4+3+a+b+2 &= 24\\
b+2+4+6+c &= 26\\
c+0+2+8+3 &= 19
\end{align}
En résolvant, on obtient la solution.\\
\textbf{Solution:}\\
\begin{tabular}{|l|l|l|}
\hline
\multicolumn{1}{|l|}{Mois} & \multicolumn{1}{l|}{Incendies} & \multicolumn{1}{l|}{Moyenne Mobile} \\ \hline
1 & 4 & \multicolumn{1}{l|}{-} \\ \hline
2 & 3 & \multicolumn{1}{l|}{-} \\ \hline
3 & \textbf{7} & 4,8 \\ \hline
4 & \textbf{8} & 4,8 \\ \hline
5 & 2 & 5,4 \\ \hline
6 & 4 & 5,2 \\ \hline
7 & 6 & 3,6 \\ \hline
8 & \textbf{6} & 3,6 \\ \hline
9 & 0 & 4,4 \\ \hline
10 & 2 & 3,8 \\ \hline
11 & 8 & \multicolumn{1}{l|}{-} \\ \hline
12 & 3 & \multicolumn{1}{l|}{-} \\ \hline
\end{tabular}
\end{sol}
\end{exercice}
\begin{exercice}
Nous sommes le 28 juin 2013, à l'heure de la fermeture des marchés
financiers. Vous possédez un titre de la compagnie BlackBerry dont la
valeur est de $S_0 = 10.46$. Un analyste vous suggère d'acheter une
option de vente européenne d'échéance de 84 jours ($t=84/365$) avec un
prix d'exercice équivalant à la valeur nominale d'un contrat à terme
de même échéance afin de couvrir le risque de baisse de la valeur de
ce titre. On considère des rendements sur une période de 28 jours et
un taux sans risque composé continument de $r=1.75\%$. En utilisant
les logarithmes des valeurs historique à la fermeture du titre
disponibles dans le fichier \url{blackberry.csv}, ainsi que la méthode
de différenciation, évaluez les rendements mensuels du titre, qui
correspondent aux résidus de ce processus différentié. Ensuite, en
évaluant la moyenne et l'écart-type de cette composante, il est
possible d'estimer la tendance linéaire $\mu$ et la volatilité
$\sigma$ mensuelles de la série des rendements. En supposant que le
prix peut prendre deux valeurs à l'échéance, soit $S_0u = S_0
e^{3(\mu+\sigma/(2\sqrt{3}))}$ et $S_0d = S_0
e^{3(\mu-\sigma/(2\sqrt{3}))}$, évaluez le prix de l'option de vente
en utilisant la probabilité neutre au risque d'une hausse $p^{*} =
\frac{e^{rt}-d}{u-d}$ et évaluez le profit que vous effectuerez en
exercant l'option de vente le 20 septembre 2013, considérant que vous
empruntez au taux $r+2\%$ pour acheter l'option.
\begin{sol}
<<>>=
rf <- 0.0175
rB <- rf+0.02
S0 <- 10.46
ST <- 8.73
K <- S0*exp(rf*84/365)
bbry <- read.csv("blackberry.csv",header=TRUE,sep=";")
bbry.sel <- bbry[as.POSIXlt(bbry$Date)$wday==5,][1+3:12*4,]$Close
l.bbry.sel <- log(bbry.sel)
(diff.l.bbry.sel <- diff(l.bbry.sel))
(mu.diff.l.bbry.sel <- mean(diff.l.bbry.sel))
(sigma.diff.l.bbry.sel <- sd(diff.l.bbry.sel))
(prix.arbre <- S0*(ud <- exp(3*(mu.diff.l.bbry.sel+c(1,-1)*
sigma.diff.l.bbry.sel/(2*sqrt(3))))))
(p.rn <- (exp(rf*84/365)-ud[2])/(ud[1]-ud[2]))
q.rn <- 1-p.rn
(P0 <- sum(exp(-rf*84/365)*(c(p.rn,q.rn)*pmax(K-prix.arbre,0))))
(BT <- P0*exp(rB*84/365))
(K-ST)-BT
@
La valeur du paramètre $\mu$ de rendement moyen est \Sexpr{round(mu.diff.l.bbry.sel,digits=4)}.
La valeur du paramètre $\sigma$ de volatilité est \Sexpr{round(sigma.diff.l.bbry.sel,digits=4)}.
La valeur des prix de l'arbre binomial sont \Sexpr{round(prix.arbre[1],digits=4)} et \Sexpr{round(prix.arbre[2],digits=4)}.
La valeur de la probabilité neutre au risque d'une hausse est \Sexpr{round(p.rn,digits=4)}.
La valeur de l'option est \Sexpr{round(P0,digits=4)}.
Le profit, qui correspont à la différence entre la réclamation contingente de l'option et le coût d'acquisition, est de \Sexpr{round((K-ST)-BT,digits=4)}.
\end{sol}
\end{exercice}
\begin{exercice}
On considère un ensemble de 10 observations:
\[
\mathcal{A}=\left\{1.2,1.5,1.4,2.1,1.8,1.9,2.2,2.4,2.0,1.9 \right\}
\]
En utilisant une méthode de lissage exponentiel avec $\alpha=0.4$ et
$\alpha=0.7$, déterminez laquelle des méthodes produit la moins grande
erreur quadratique moyenne (MSE).
\begin{sol}
On calcule d'abord les deux séries lissées \\
\begin{tabular}{|l|r|r|}
\hline
\multicolumn{1}{|c|}{$\mathcal{A}$} & $\alpha=0,4$ & $\alpha=0,7$ \\ \hline
1,2 & 1,2000 & 1,2000 \\ \hline
1,5 & 1,3200 & 1,4100 \\ \hline
1,4 & 1,3520 & 1,4030 \\ \hline
2,1 & 1,6512 & 1,8909 \\ \hline
1,8 & 1,7107 & 1,8273 \\ \hline
1,9 & 1,7864 & 1,8782 \\ \hline
2,2 & 1,9519 & 2,1035 \\ \hline
2,4 & 2,1311 & 2,3110 \\ \hline
2,0 & 2,0787 & 2,0933 \\ \hline
1,9 & 2,0072 & 1,9580 \\ \hline
\end{tabular} \\
On évalue ensuite l'erreur quadratique pour chaque terme \\
\begin{tabular}{|l|r|r|}
\hline
\multicolumn{1}{|c|}{$\mathcal{A}$} & \multicolumn{1}{l|}{$SE(\alpha=0,4)$} & \multicolumn{1}{l|}{$SE(\alpha=0,7)$} \\ \hline
1,2 & \multicolumn{1}{l|}{} & \multicolumn{1}{l|}{} \\ \hline
1,5 & 0,0324 & 0,0081 \\ \hline
1,4 & 0,0023 & 0,0000 \\ \hline
2,1 & 0,2014 & 0,0437 \\ \hline
1,8 & 0,0080 & 0,0007 \\ \hline
1,9 & 0,0129 & 0,0005 \\ \hline
2,2 & 0,0616 & 0,0093 \\ \hline
2,4 & 0,0723 & 0,0079 \\ \hline
2,0 & 0,0062 & 0,0087 \\ \hline
1,9 & 0,0115 & 0,0034 \\ \hline
\end{tabular} \\
On obtient enfin l'erreur quadratique moyenne\\
\begin{tabular}{|l|l|l|}
\hline
& $\alpha=0,4$ & $\alpha=0,7$ \\ \hline
MSE & \multicolumn{1}{r|}{0,0454} & \multicolumn{1}{r|}{0,0092} \\ \hline
\end{tabular} \\
Les calculs effectués se trouvent dans le fichier \url{Lissage.Exponentiel.I.ods} \footnote{Ce fichier est au format OpenDocument et s'ouvre avec la plupart des suites bureautiques}.
Avec R, on obtient les résultats suivants en utilisant la fonction de lissage exponentiel \emph{EMA()}.
<<>>=
A <- c(1.2, 1.5, 1.4, 2.1, 1.8, 1.9, 2.2, 2.4, 2.0, 1.9)
n.A <- length(A)
A.EMA.4 <- EMA(A,n=1,ratio=0.4)
A.EMA.7 <- EMA(A,n=1,ratio=0.7)
A.SE <- (A-cbind(A.EMA.4,A.EMA.7))^2
cbind(A,A.EMA.4,A.EMA.7,A.SE)
(A.MSE <- colMeans(A.SE)*(n.A/(n.A-1)))
@
La valeur $\alpha=0.7$ produit une erreur quadratique moyenne inférieure.
\end{sol}
\end{exercice}
\begin{exercice}
En utilisant les données du problème précédent, déterminez, à l'aide
d'un algorithme informatique, la valeur de $\alpha$ comprise entre $0.4$ et $0.7$ qui minimise l'erreur quadratique moyenne (MSE) pour un lissage exponentiel double.
\begin{sol}
Une solution assez simple est d'utiliser le solveur intégré au logiciel tableau que vous utilisez et d'optimiser la valeur de la cellule contenant $\alpha$ avec comme critère de minimisation la cellule contenant l'erreur quadratique moyenne (MSE).
On peut aussi construire une fonction d'optimisation dans R qui réplique le comportement du chiffrier que nous avons construit dans le logiciel tableur.
<<>>=
funOptAlphaDEMA <- function(alpha,data)
{
data.n <- length(data)
data.DEMA <- DEMA(A,n=1,ratio=alpha)
data.SE <- (data-data.DEMA)^2
data.MSE <- mean(data.SE)*data.n/(data.n-1)
print(c(data.MSE,alpha))
data.MSE
}
optimize(funOptAlphaDEMA,c(0.4,0.7),A)
@
Tout comme pour le lissage exponentiel effectué à la question précédente, la valeur $\alpha=0.7$ produit une erreur quadratique moyenne inférieure.
\end{sol}
\end{exercice}
\begin{exercice}
On considère les 20 observations de la valeur ajustée à la fermeture (valeur qui tient compte des dividendes) du titre de la Bank of America pour chaque lundi entre le 20 mai 2013 et le 30 septembre 2013. Ces données se trouvent dans le fichier \url{BoA.csv}.
\begin{enumerate}
\item En utilisant le test du corrélogramme avec un seuil de tolérance de $\alpha = 10\%$, déterminez s'il s'agit d'une série stationnaire (Bruit blanc).
\item En utilisant le test du changement de direction avec un seuil de tolérance de $\alpha = 10\%$, déterminez s'il s'agit d'une série stationnaire.
\item En utilisant le test de Portmanteau avec un seuil de tolérance de $\alpha = 10\%$, déterminez s'il s'agit d'une série stationnaire.
\item Est-ce que ces tests sont équivalents ? Commentez.
\item En utilisant la différenciation et le logarithme des données, évaluez la série des rendements hebdomadaires.
\item En utilisant le même principe que pour la moyenne mobile, évaluez la variance mobile de la série précédente avec $q=2$. Que remarquez-vous ? Peut-on affirmer que c'est une série stationnaire à l'aide du test de Portmanteau avec un seuil de tolérance de $\alpha = 10\%$?
\end{enumerate}
\begin{sol}
On importe d'abord l'ensemble de données
<<>>=
BoA <- ts(read.csv("BoA.csv",header=TRUE,sep="\t"))
@
\begin{enumerate}
\item
On trace ensuite le corrélogramme (figure \ref{fig:exercice1.6-graph1})
La fonction \emph{acf} nous permet d'afficher un corrélogramme
<<echo=FALSE>>=
pdf("exercice1-6-graph1.pdf",paper="special", width=6,height=6)
acf(BoA[,2],lag.max=19)
dummy <- dev.off()
@
\begin{figure}[!ht]
\centering
\includegraphics[height=4in, width=4in]{exercice1-6-graph1}
\caption{Corrélogramme de la série BoA}
\label{fig:exercice1.6-graph1}
\end{figure}
La fonction d'autocorrélation empirique $\hat{\rho}$ prend les valeurs
suivantes:
<<>>=
(BoA.acf <- acf(BoA[,2],lag.max=19))
dummy <- dev.off()
@
En utilisant la méthode vue dans le cours, on construit un intervalle de
confiance au niveau $1-\alpha=0.9$ à partir de la distribution normale. La
valeur de $n$ est \Sexpr{length(BoA[,2])}.
<<>>=
(BoA.acf.IC <- round(c(1/sqrt(20)*qnorm(0.05),-1/sqrt(20)*qnorm(0.05)),4))
@
\begin{align*}
IC &= \frac{1}{\sqrt{n}}\left[-z_{\alpha / 2},z_{\alpha / 2} \right] \\
&= \frac{1}{\sqrt{20}}\left[-z_{0.05},z_{0.05} \right] \\
&= \left[ \Sexpr{BoA.acf.IC[1]},\Sexpr{BoA.acf.IC[2]} \right]
\end{align*}
<<>>=
(BoA.nbacfplus <- sum(BoA.acf$acf[-1]<BoA.acf.IC[1]) +
sum(BoA.acf$acf[-1]>BoA.acf.IC[2]))
@
Comme \Sexpr{BoA.nbacfplus} valeurs sur 20, soit 10\% de celles-ci, sont à l'extérieur de
l'intervalle de confiance, alors on ne peut rejeter l'hypothèse selon laquelle la
série est stationnaire lorsqu'on se base sur le test du corrélogramme.
\item
Ici, on n'a qu'à tracer la série et compter les changements de direction
(figure \ref{fig:exercice1.6-graph2})
<<echo=FALSE>>=
pdf("exercice1-6-graph2.pdf",paper="special", width=6,height=6)
plot(BoA[,2])
dummy <- dev.off()
@
\begin{figure}[!ht]
\centering
\includegraphics[height=4in, width=4in]{exercice1-6-graph2}
\caption{Corrélogramme de la série BoA}
\label{fig:exercice1.6-graph2}
\end{figure}
On en dénombre 9.
<<>>=
BoA.chdir <- abs((9-(2/3)*18)/sqrt((16*20-29)/90))
BoA.chdir > qnorm(0.95)
@
On évalue la statistique de test, qui prend la valeur \Sexpr{round(BoA.chdir,4)}. Comme cette valeur est supérieure au seuil de \Sexpr{round(qnorm(0.95),4)}, on rejette l'hypothèse de stationnarité avec le test du changement de direction.
\item
Il existe deux tests de type Portmanteau. Celui que vous avez vu en classe est le test de Box-Pierce où h commence à 1.
<<>>=
Box.test(BoA[,2],lag=19,type="Box-Pierce")
qchisq(0.9,19)
@
On rejette l'hypothèse de stationnarité car la valeur de $Q^{*}=33.5182$ est supérieure au quantile $\chi^2_{0.1}(19) = 27.20357$
\item
Les tests sont indépendants, différents entre eux et ne sont pas équivalents
car leurs statistiques ne suivent pas la même distribution asymptotique.
\item
<<>>=
round(diff(log(BoA[,2])),4)
@
\item
<<>>=
(BoA.hist.var <- zoo::na.trim(apply(cbind(BoA[,2],
lag(BoA[,2],1),
lag(BoA[,2],2),
lag(BoA[,2],3),
lag(BoA[,2],4)),
1,
var)))
Box.test(BoA.hist.var,lag=15,type="Box-Pierce")
qchisq(0.9,15)
@
On remarque que la série des volatilités historiques avec $q=2$ est stationnaire bien que la volatilité ne soit pas constante.
\end{enumerate}
\end{sol}
\end{exercice}
\begin{exercice}
Une série présentant une racine unitaire se présente sous la forme $Y_t = Y_{t-1}+\epsilon_t$. Quelle est la différence entre la variance du $5^e$ terme et du $7^e$ terme de cette série si $\epsilon_t \sim N(0, 0.1t^2)$?
\begin{sol}
La variance du $t^e$ terme est équivalente à la somme de la variance des $t$ premiers termes d'erreurs. La différence entre les variances des $5^e$ terme et du $7^e$ terme est donc égale à la somme:
\begin{align*}
\label{eq:2}
V\left[\epsilon_6\right]+V\left[\epsilon_7\right] &= 0.1(6^2+7^2) \\
& = 8.5
\end{align*}
\end{sol}
\end{exercice}
\Closesolutionfile{solutions}
\Closesolutionfile{reponses}
%%% Local Variables:
%%% mode: latex
%%% TeX-master: t
%%% End:

551
serie2-exercices.Rnw Normal file
View file

@ -0,0 +1,551 @@
\chapter{Modèles classiques pour séries chronologiques}
\label{chap:modeles-classiques}
\Opensolutionfile{reponses}[reponses-modeles-classiques]
\Opensolutionfile{solutions}[solutions-modeles-classiques]
\begin{Filesave}{reponses}
\bigskip
\section*{Réponses}
\end{Filesave}
\begin{Filesave}{solutions}
\section*{Chapitre \ref{chap:modeles-classiques}}
\addcontentsline{toc}{section}{Chapitre \protect\ref{chap:modeles-classiques}}
\end{Filesave}
<<echo=FALSE>>=
options(width = 55)
@
\begin{exercice}
Pour avoir la stationnarité, il faut que les racines du polynôme caractéristique soient inférieures à 1 en valeur absolue. Démontrez que pour le modèle AR(2), la stationnarité est possible si et seulement si les trois conditions suivantes sont réunies:
\begin{align*}
\phi_1 + \phi_2 &< 1 \\
\phi_2 - \phi_1 &< 1 \\
|\phi_2| &< 1
\end{align*}
\begin{sol}
On obtient les racines de l'équation caractéristique en utilisant la formule quadratique habituelle
\begin{align*}
\frac{\phi_1\pm\sqrt{\phi_1^2+4\phi_2}}{-2\phi_2}
\end{align*}
On considère les inverses des deux racines, $A_1$ et $A_2$.
\begin{align*}
A_1 &= \frac{2\phi_2}{-\phi_1-\sqrt{\phi_1^2+4\phi_2}} \\
&= \frac{2\phi_2}{-\phi_1-\sqrt{\phi_1^2+4\phi_2}} \left[\frac{-\phi_1+\sqrt{\phi_1^2+4\phi_2}}{-\phi_1+\sqrt{\phi_1^2+4\phi_2}} \right] \\
&= \frac{2\phi_2(-\phi_1+\sqrt{\phi_1^2+4\phi_2})}{\phi_1^2-(\phi_1^2+4\phi_2)}\\
&= \frac{\phi_1-\sqrt{\phi_1^2+4\phi_2}}{2}\\
A_2 &= \frac{\phi_1+\sqrt{\phi_1^2+4\phi_2}}{2}
\end{align*}
Il y a 2 situations possibles: soit les racines sont réelles ($\phi_1^2+4\phi_2>0$) ou elles sont complexes ($\phi_1^2+4\phi_2<0$).
\begin{itemize}
\item \textbf{Racines réelles:}
Comme les racines doivent être plus grandes que 1, alors nécessairement leurs inverses $|A_1|<1$ et $|A_2|<1$. Nous avons donc:
\begin{align*}
-1 &< \frac{\phi_1-\sqrt{\phi_1^2+4\phi_2}}{2} < \frac{\phi_1+\sqrt{\phi_1^2+4\phi_2}}{2} < 1 \\
\Leftrightarrow -2 &< \phi_1-\sqrt{\phi_1^2+4\phi_2} < \phi_1+\sqrt{\phi_1^2+4\phi_2} < 2
\end{align*}
En observant la première inégalité, on a:
\begin{align*}
-2 < \phi_1-\sqrt{\phi_1^2+4\phi_2}
&\Leftrightarrow \sqrt{\phi_1^2+4\phi_2}<\phi_1+2 \\
&\Leftrightarrow \phi_1^2+4\phi_2 < \phi_1^2+4\phi_1+4 \\
&\Leftrightarrow \phi_2 < \phi_1 + 1 \\
&\Leftrightarrow \phi_2 - \phi_1 < 1
\end{align*}
Ce qui correspond à la seconde condition. En considérant la seconde inégalité, on obtient, de la même façon, la première inégalité:
\begin{align*}
\phi_1+\sqrt{\phi_1^2+4\phi_2} &< 2 \\
&\Leftrightarrow \phi_1 + \phi_2 < 1
\end{align*}
Ces deux conditions réunies avec un discriminant positif forment la région de stationnarité pour des racines réelles.
\item \textbf{Racines complexes:}
On considère la situation où $\phi_1^2+4\phi_2<0$. Ici, on aura des conjugués complexes et $|A_1| = |A_2| <1$ seulement si $|A_1|^2<1$.
\begin{align*}
|A_1|^2 &=\frac{\phi_1^2+(-\phi_1^2-4\phi_2}{4}=-\phi^2 \\
&\Leftrightarrow \phi_2>-1 \\
&\Leftrightarrow |\phi_2|<1
\end{align*}
Ce résultat réuni avec un discriminant négatif forment la région de stationnarité pour des racines complexes.
\end{itemize}
\end{sol}
\end{exercice}
\begin{exercice}
\begin{enumerate}
\item
Un polynôme d'ordre $k$ en t est intégré d'ordre $k$ puisque
\begin{align*}
(1-B)^k(a_0+a_1t+a_2t^2+\ldots+a_kt^k) &= k!a_k
\end{align*}
Démontrez cette affirmation.
\item
Démontrez que si $x_t$ est stationnaire, alors $(1-B)x_t$ est aussi stationnaire.
\end{enumerate}
\begin{sol}
\begin{enumerate}
\item On obtient ce résultat par récurrence. Par exemple, pour $k=2$, on a :
\begin{align*}
(1-B)^2 (a_0+a_1t+a_2t^2) &= (1-B) ((a_0+a_1t+a_2t^2)\\ &\quad- (a_0+a_1(t-1)+a_2(t-1)^2)) \\
&= (1-B) (a_1+a_2(2t+1)) \\
&= (a_1+a_2(2t+1)) - (a_1+a_2(2(t-1)+1)) \\
&= 2a_2
\end{align*}
En général, on obtient:
\begin{align*}
(1-B)^k (a_0+a_1t+a_2t^2+\ldots+a_kt^k) &= (1-B)^{k-1} ((a_0+a_1t+a_2t^2+\ldots+a_kt^k)\\ &\quad- (a_0+a_1(t-1)+a_2(t-1)^2+\ldots+a_k(t-1)^k)) \\
&= (1-B)^{k-1} (a_1 + 2a_2t+\ldots+a_k(t^k-(t-1)^k))
\end{align*}
On remarque qu'à chaque itération, le premier terme de la série disparait. Ainsi, après $k$ itérations, il ne restera que le terme en $a_k$ avec son coefficient, qui correspont à $k!$. On obtient ainsi la solution générale.
\item Une série est dite stationnaire lorsque chaque terme est un terme d'erreur dont la distribution est constante au fil du temps. Ainsi, la distribution de la différence de deux termes consécutifs de la série sera aussi constante au fil du temps. Par exemple:
\begin{align*}
\epsilon_t \sim N(0,\sigma^2) \\
\epsilon_t - \epsilon_{t-1} \sim N(0,2\sigma^2) \\
\end{align*}
\end{enumerate}
\end{sol}
\end{exercice}
\begin{exercice}
\begin{enumerate}
\item
Démontrez algébriquement qu'un processus AR(1) est équivalent à un processus MA($\infty$).
\item
Démontrez algébriquement qu'un processus MA(1) est équivalent à un processus AR($\infty$).
\end{enumerate}
\begin{sol}
\begin{enumerate}
\item Un processus AR(1) est défini par $y_t = \phi_1y_{t-1} + \epsilon_t$. En développant le terme $y_{t-1}$, on obtient $y_t = \phi_1^2y_{t-2}+\phi_1\epsilon_{t-1}+\epsilon_t$. De manière récursive, on obtient $y_t = \phi_1^{t}\epsilon_0 + \phi_1^{t-1}\epsilon_1 + \ldots + \phi_1\epsilon_{t-1} + \epsilon_t$. ainsi, en faisant tendre $t\to\infty$, on obtient une représentation MA($\infty$).
\item Un processus MA(1) est défini par $y_t = \epsilon_t - \theta_1\epsilon_{t-1}$. On cherche à substituer le terme $\epsilon_{t-1}$. On développe le terme précédent de la série: $y_{t-1} = \epsilon_{t-1} - \theta_1\epsilon_{t-2}$ et on substitue dans la première expression pour obtenir $y_t = \epsilon_t - \theta_1y_{t-1} - \theta_1^2\epsilon_{t-2}$. De manière récursive, on obtient $y_t = -\theta_1^ty_0-\theta_1^{t-1}y_1-\ldots-\theta_1y_{t-1}+\epsilon_{t}$. Lorsque $t\to\infty$, on obtient une représentation AR($\infty$).
\end{enumerate}
\end{sol}
\end{exercice}
\begin{exercice}
On considère les 10 nombres aléatoires suivants, issus d'une distribution normale centrée réduite:
\begin{verbatim}
[1] -1.21 0.28 1.08 -2.35 0.43 0.51 -0.57 -0.55 -0.56 -0.89
\end{verbatim}
Construisez la série autorégressive d'ordre 1 avec coefficient:
\begin{enumerate}
\item $\phi = -0.5$
\item $\phi = 0.5$
\end{enumerate}
Quelle différence observez-vous entre la série avec une corrélation négative et la série avec une corrélation positive ?
\begin{sol}
On utilise la formule $y_t = \phi_1y_{t-1} + \epsilon_t$.
\begin{center}
\begin{tabular}{|r|r|r|}
\hline
\multicolumn{1}{|l|}{} & \multicolumn{ 2}{c|}{$\phi$} \\ \hline
\multicolumn{1}{|l|}{$N(0,1)$} & -0,5 & 0,5 \\ \hline
-1,21 & -1,2100 & -1,2100 \\ \hline
0,28 & 0,8850 & -0,3250 \\ \hline
1,08 & 0,6375 & 0,9175 \\ \hline
-2,35 & -2,6688 & -1,8913 \\ \hline
0,43 & 1,7644 & -0,5156 \\ \hline
0,51 & -0,3722 & 0,2522 \\ \hline
-0,57 & -0,3839 & -0,4439 \\ \hline
-0,55 & -0,3580 & -0,7720 \\ \hline
-0,56 & -0,3810 & -0,9460 \\ \hline
-0,89 & -0,6995 & -1,3630 \\ \hline
\end{tabular}
\end{center}
Les séries avec une corrélation négative ont tendance à aller dans la direction contraire des termes précédents alors que celles avec une corrélation positive ont tendance à aller dans la même direction que les termes précédents.
Le tableur \url{constructionserieAR.ods} contient les calculs effectués.
\end{sol}
\end{exercice}
\begin{exercice}
On considère deux processus MA(2), un où $\theta_1 = \theta_2 = \frac{1}{4}$, et un autre où $\theta_1=-1$ et $\theta_2 = 4$. Démontrez que ces processus ont la même fonction d'autocorrélation.
\begin{sol}
Il suffit de calculer la fonction d'autorégression pour chaque processus MA(2).
Ensuite, on peut évaluer la fonction d'autocorrélation et comparer le résultat obtenu.
\begin{enumerate}
\item Premier processus avec:
\begin{align*}
\theta_1 = \theta_2 = \frac{1}{4}
\end{align*}
Fonction d'autocovariance
\begin{align*}
\phi_0^{(1)} &= V[Y_t] \\
&= V[e_t]+\frac{1}{16}V[e_{t-1}]+\frac{1}{16}V[e_{t-2}] \\
&= (1+\frac{1}{8})\theta^2_e \\
&= \frac{9}{8} \sigma^2_e \\
\end{align*}
\begin{align*}
\phi_1^{(1)} &= Cov(Y_t,Y_{t-1}) \\
&= Cov(e_t - \frac{1}{4}e_{t-1} - \frac{1}{4}e_{t-2}, e_{t-1} - \frac{1}{4}e_{t-2} - \frac{1}{4}e_{t-3} )\\
&= Cov(-\frac{1}{4}e_{t-1},e_{t-1}) + Cov(-\frac{1}{4}e_{t-2},-\frac{1}{4}e_{t-2}) \\
&= (-\frac{1}{4}+(-\frac{1}{4})(-\frac{1}{4})) \sigma^2_e \\
&= -\frac{3}{16} \sigma^2_e \\
\end{align*}
\begin{align*}
\phi_2^{(1)} &= Cov(Y_t,Y_{t-2}) \\
&= Cov(e_t - \frac{1}{4}e_{t-1} - \frac{1}{4}e_{t-2}, e_{t-2} - \frac{1}{4}e_{t-3} - \frac{1}{4}e_{t-4} )\\
&= Cov(-\frac{1}{4}e_{t-2},e_{t-2}) \\
&= -\frac{1}{4} \sigma^2_e \\
\end{align*}
\begin{align*}
\phi_k^{(1)} &= 0, \qquad \forall k \geq 3
\end{align*}
Fonction d'autocorrélation
\begin{align*}
\rho_1^{(1)} &= \frac{\phi_1^{(1)}}{\phi_0^{(1)}} \\
&= \frac{\frac{-3}{16}\sigma^2_e}{\frac{9}{8}\sigma^2_e} \\
&= \frac{-1}{6} \\
\end{align*}
\begin{align*}
\rho_2^{(1)} &= \frac{\phi_2^{(1)}}{\phi_0^{(1)}} \\
&= \frac{\frac{-1}{4}\sigma^2_e}{\frac{9}{8}\sigma^2_e} \\
&= \frac{-2}{9} \\
\end{align*}
\item Second processus avec:
\begin{align*}
\theta_1 = -1 \quad \theta_2 = 4
\end{align*}
Fonction d'autocovariance
\begin{align*}
\phi_0^{(2)} &= V[Y_t] \\
&= V[e_t]+V[e_{t-1}]+16V[e_{t-2}] \\
&= 18 \sigma^2_e
\end{align*}
\begin{align*}
\phi_1^{(2)} &= Cov(Y_t,Y_{t-1}) \\
&= Cov(e_t + e_{t-1} - 4e_{t-2}, e_{t-1} + e_{t-2} - 4e_{t-3}) \\
&= Cov(e_{t-1},e_{t-1}) + Cov(-4e_{t-2},e_{t-2}) \\
&= (1 + (-1)(4))\sigma^2_e \\
&= -3\sigma^2_e \\
\end{align*}
\begin{align*}
\phi_2^{(2)} &= Cov(Y_t,Y_{t-2}) \\
&= Cov(e_t + e_{t-1} - 4e_{t-2}, e_{t-2} + e_{t-3} - 4e_{t-4} )\\
&= Cov(- 4e_{t-2},e_{t-2}) \\
&= -4\sigma^2_e \\
\end{align*}
Fonction d'autocorrélation
\begin{align*}
\rho_1^{(2)} &= \frac{\phi_1^{(2)}}{\phi_0^{(2)}} \\
&= \frac{-3\sigma^2_e}{18\sigma^2_e} \\
&= \frac{-1}{6} \\
\end{align*}
\begin{align*}
\rho_2^{(2)} &= \frac{\phi_2^{(2)}}{\phi_0^{(2)}} \\
&= \frac{-4\sigma^2_e}{18\sigma^2_e} \\
&= \frac{-2}{9} \\
\end{align*}
\end{enumerate}
On remarque clairement que $\rho_1^{(1)} = \rho_1^{(2)}$ et $\rho_2^{(1)} = \rho_2^{(2)}$.
La fonction d'autocovariance vaut toujours 1 pour $\rho_1$ et vaut 0 ailleurs.
\end{sol}
\end{exercice}
\begin{exercice}
\begin{enumerate}
\item En utilisant les équations de Yule-Walker, dérivez un estimateur des moments pour les paramètres $\phi_1$ et $\phi_1$ d'un processus AR(2). \\
\item Estimez les paramètres du processus AR(2) à partir de la série suivante:
\begin{verbatim}
[1] 1.1617660 0.6981185 0.1693004 -0.6457205 1.4217278 1.3701445
[7] -1.6369769 -0.4596686 -0.2933815 -1.0995973
\end{verbatim}
\end{enumerate}
\begin{sol}
\begin{enumerate}
\item
Les deux équations de Yule-Walker pour le modèle AR(2) sont les suivantes:
\begin{align*}
\rho_1 = \phi_1 + \rho_1 \phi_2 \\
\rho_2 = \rho_1\phi_1 + \phi_2
\end{align*}
En utilisant l'estimateur de la fonction d'autocovariance $\hat{\rho}$, on obtient alors:
\begin{align*}
\hat{\phi_1} = \frac{\hat{\rho_1}(1-\hat{\rho_2})}{1-\hat{\rho_1}^2} \\
\hat{\phi_2} = \frac{\hat{\rho_2} - \hat{\rho_1}^2}{1-\hat{\rho_1}^2}
\end{align*}
\item
<<>>=
set.seed(123)
(serie <- arima.sim(n = 10, list(ar = c(0.5,-0.25))))
acf.serie <- acf(serie,type="correlation",plot=FALSE)$acf[2:3]
phi1 <- acf.serie[1]*(1-acf.serie[2]) / (1-acf.serie[1]^2)
phi2 <- (acf.serie[2] - acf.serie[1]^2) / (1-acf.serie[1]^2)
@
On obtient $\hat{\rho_1} = \Sexpr{round(acf.serie[1],5)}$ et $\hat{\rho_2} = \Sexpr{round(acf.serie[2],5)}$.
Ce qui nous donne les paramètres du modèle AR(2) suivants:
$\hat{\phi_1} = \Sexpr{round(phi1,5)}$ et
$\hat{\phi_2} = \Sexpr{round(phi2,5)}$.
\end{enumerate}
\end{sol}
\end{exercice}
\begin{exercice}
\begin{enumerate}
\item Démontrez que le terme d'erreur $\epsilon_t$ d'un processus ARMA(2,1) peut être exprimé sous la forme suivante, où $\mu$ est une constante et $\phi_1, \phi_2, \theta$ sont les paramètres du modèle. On considère que la série est stationnaire.
\begin{align*}
\epsilon_t = \sum_{i=0}^{\infty} \theta^i \left(y_{t-i} - \mu - \phi_1 y_{t-i-1} - \phi_2 y_{t-i-2} \right)
\end{align*}
\item De plus, démontrez qu'à partir de cette forme du terme d'erreur, on peut obtenir la représentation AR($\infty$) du processus ARMA(2,1).
\end{enumerate}
\begin{sol}
\begin{enumerate}
\item
On représente le processus ARMA(2,1) sous la forme suivante:
\begin{align*}
y_t = \mu + \phi_1 y_{t-1} + \phi_2 y_{t-2} + \epsilon_t - \theta\epsilon_{t-1}
\end{align*}
En utilisant l'opérateur de rétrodécalage $B$, on peut exprimer cette équation sous la forme suivante:
\begin{align*}
(1-\theta B)\epsilon_t = y_t - \mu - \phi_1 y_{t-1} - \phi_2 y_{t-2}
\end{align*}
On divise ensuite de chaque côté par $(1-\theta B)$, pour obtenir:
\begin{align*}
\epsilon_t = \frac{1}{(1-\theta B)} \left(y_t - \mu - \phi_1 y_{t-1} - \phi_2 y_{t-2}\right)
\end{align*}
L'hypothèse de stationnarité nous permet de poser que $| \theta | < 1$, ce qui nous permet d'utiliser la série géométrique définie comme suit:
\begin{align*}
\frac{1}{1-\theta B} = \sum_{i=0}^{\infty} \theta^i B^i
\end{align*}
On obtient donc que
\begin{align*}
\epsilon_t = \sum_{i=0}^{\infty} \theta^i B^i \left(y_t - \mu - \phi_1 y_{t-1} - \phi_2 y_{t-2}\right)
\end{align*}
En appliquant l'opérateur de rétrodécalage à la parenthèse, on obtient la solution:
\begin{align*}
\epsilon_t = \sum_{i=0}^{\infty} \theta^i \left(y_{t-i} - \mu - \phi_1 y_{t-i-1} - \phi_2 y_{t-i-2} \right)
\end{align*}
\item
On exprime l'équation précédente en fonction de $y_t$:
\begin{align*}
y_t &= \phi_1 y_{t-1} + \phi_2 y_{t-2} - \sum_{i=1}^{\infty} \theta^i \left(y_{t-i} - \phi_1 y_{t-i-1} - \phi_2 y_{t-i-2} \right) + \epsilon_t + \frac{\mu}{1-\theta} \\
&= \phi_1 y_{t-1} + \phi_2 y_{t-2} - \left[\theta\left(y_{t-1}-\phi_1 y_{t-2} - \phi_2 y_{t-3} \right) + \sum_{i=2}^{\infty} \theta^i \left(y_{t-i} - \phi_1 y_{t-i-1} - \phi_2 y_{t-i-2} \right)\right] + \epsilon_t + \frac{\mu}{1-\theta} \\
&= (\phi_1 - \theta) y_{t-1} - \sum_{i=2}^{\infty} \left(\theta_i+\phi_1\theta^{i-1} - \phi_2\theta_{i-2}\right) y_{t-i} + \epsilon_t + \frac{\mu}{1-\theta}
\end{align*}
Ce qui correspont à la forme autorégressive AR($\infty$) suivante:
\begin{align*}
y_t = c + \sum_{i=1}^{\infty} \pi_i y_{t-i} + \epsilon_t
\end{align*}
Avec les paramètres
\begin{align*}
c &= \frac{\mu}{1-\theta} \\
\pi_1 &= (\phi_1 - \theta) \\
\pi_i &= -\left(\theta_i+\phi_1\theta^{i-1} - \phi_2\theta_{i-2}\right),\quad i=2,3,\ldots
\end{align*}
\end{enumerate}
\end{sol}
\end{exercice}
\begin{exercice}
On considère l'équation en différence suivante:
\begin{align*}
y_t = 1.5 y_{t-1} - 0.5 y_{t-2} + \epsilon_t
\end{align*}
\begin{enumerate}
\item À quel modèle correspond cette équation ?
\item Trouvez les racines de l'équation homogène.
\item Démontrez que les racines de l'équation $1-1.5B+0.5B^2$ sont la réciproque des valeurs trouvées à la question précédente.
\item Est-ce que cette série est stationnaire ?
\item On suppose que l'on connaît les deux premiers termes de la série $y_0$ et $y_1$. Trouvez la solution générale pour $y_t$ en fonction de la séquence des valeurs de $\epsilon_t$.
\item Identifiez la forme de la fonction de prédiction pour $y_{T+s}$, sachant les valeurs de $y_{T-1}$ et $y_T$.
\item Évaluez $E[y_t]$, $E[y_{t+1}]$, $Var[y_t]$, $Var[y_{t+1}]$ et $Cov[y_{t},y_{t+1}]$.
\item Donnez l'expression d'un intervalle de confiance à 95\% pour la valeur de $y_{t+1}$
\end{enumerate}
\begin{sol}
\begin{enumerate}
\item C'est un modèle AR(2) de paramètres $\phi_1=1.5$ et $\phi_2=-0.5$.
\item L'équation caractéristique prend la forme suivante:
\begin{align*}
y_t - 1.5 y_{t-1} + 0.5y_{t-2} = 0
\end{align*}
On pose la solution générale $y_t=A\alpha^t$:
\begin{align*}
A\alpha^t - 1.5 A\alpha^{t-1} + 0.5A\alpha^{t-2} = 0
\end{align*}
On divise ensuite par $A\alpha^{t-2}$:
\begin{align*}
0 &= \alpha^2 - 1.5 \alpha + 0.5\\
\alpha &= \frac{1.5 \pm \sqrt{1.5^2-4*1*0.5}}{2} \\
&= \left\{0.5 ; 1 \right\}
\end{align*}
\item
\begin{align*}
1 - 1.5B + 0.5B^2 &= 0 \\
(1-B)(1-0.5B) &= 0
\end{align*}
Ce qui nous donne:
\begin{align*}
B &= \left\{ 1;2 \right\} \\
B^{-1} &= \left\{ 1;0.5 \right\}
\end{align*}
\item Nous n'avons pas la stationnarité, car $\alpha$ n'est pas à l'intérieur du cercle unité
\item
\begin{align*}
y_2 &= 1.5 y_1 - 0.5y_0 + \epsilon_2 \\
y_3 &= 1.5 y_2 - 0.5y_1 + \epsilon_3 \\
&= \epsilon_3 + 1.5 \epsilon_2 + 1.75y_1 - 0.75 y_0 \\
y_4 &= 1.5 y_3 - 0.5y_2 + \epsilon_4 \\
&= \epsilon_4 + 1.5 \epsilon_3 + 1.75 \epsilon_2 + 1.875 y_1 - 0.875 y_0
\end{align*}
On peut observer un motif répétifif, que l'on représente sous la forme
\begin{align*}
y_t &= \sum_{i=0}^{t-2} \alpha_i\epsilon_{t-i} + \alpha_{t-1}y_1 + \alpha_t y_0 \\
\end{align*}
\begin{align*}
\alpha_0 &= 1\\
\alpha_1 &= 1.5\\
\alpha_t &= 1-\alpha_{t-1}
\alpha_i &= 1.5 \alpha_{i-1} - 0.5 \alpha_{i-2}
\end{align*}
\item
\begin{align*}
y_s &= \sum_{i=0}^{s-2} \alpha_i\epsilon_{s-i} + \alpha_{s-1}y_1 + \alpha_s y_0 \\
y_{t+s} &= \sum_{i=0}^{s-2} \alpha_i\epsilon_{t+s-i} + \alpha_{s-1}y_{t+1} + \alpha_s y_t \\
E_{t+1}\left[y_{t+s}\right] &= \alpha_{s-1} y_{t+1} + \alpha_s y_t
\end{align*}
\item
\begin{align*}
E\left[y_t\right] &= \alpha_{t-1} y_1 + \alpha_t y_0 \\
E\left[y_{t+1}\right] &= \alpha_{t} y_1 + \alpha_{t+1} y_0 \\
Var\left[y_t\right] &= \left[1+\alpha_1^2 + \alpha_2^2 + \ldots + \alpha_{t-2}^2\right]\sigma^2 \\
Var\left[y_{t+1}\right] &= Var\left[y_t\right] + \alpha_{t-1}^2 \sigma^2 \\
Cov\left[y_{t},y_{t+1}\right] &= \left[\alpha_0\alpha_1 + \alpha_1\alpha_2 + \ldots + \alpha_{t-3}\alpha_{t-2}\right] \sigma^2
\end{align*}
\item
\begin{align*}
\alpha_{t} y_1 + \alpha_{t+1} y_0 \pm 1.96 * \sigma \sqrt{1+\alpha_1^2 + \alpha_2^2 + \ldots + \alpha_{t-2}^2+\alpha_{t-1}^2}
\end{align*}
\end{enumerate}
\end{sol}
\end{exercice}
\begin{exercice}
Soit les deux processus suivants:
\begin{align*}
Y_t &= V_t + \alpha V_{t-1} \\
Z_t &= \delta_t
\end{align*}
On ajoute que $V_t$ et $\delta_t$ sont indépendants.
\begin{enumerate}
\item Déterminer le modèle classique pour le processus $X_t = Y_t + Z_t$ sous la forme ARMA(p,q).
\item Évaluer les paramètres $\theta_1,\ldots,\theta_n, \phi_1, \ldots \phi_n$ et $\sigma^2_{\epsilon}$ du modèle identifié précédemment, considérant que $\alpha=0.5, \sigma^2_{V}=0.04$ et $\sigma^2_{\delta}=0.01$.
\end{enumerate}
\begin{sol}
\begin{enumerate}
\item
\begin{align*}
Var[X_t] &= Var[Y_t] + Var[Z_t] \\
&= (1+\alpha^2)\sigma^2_{V} + \sigma^2_{\delta} \\
E[X_t,X_{t-1}] &= E[(v_t+\delta_t+\alpha v_{t-1})(v_{t-1}+\delta_{t-1}+\alpha v_{t-2})] \\
&= \alpha \sigma^2_{V} \\
E[X_t,X_{t-k}] &= 0, \quad\forall k \neq \{ 0,1 \} \\
&\Rightarrow \gamma_0,\gamma_1 \neq 0; \gamma_k=0 \forall k \neq \{ 0,1 \} \\
\end{align*}
Par conséquent, il s'agit d'un modèle MA(1).
\item
\begin{align*}
\gamma_0 &= (1+\theta_1^2) \sigma^2_{\epsilon} = (1+\alpha^2)\sigma^2_{V} + \sigma^2_{\delta} \\
\gamma_1 &= \theta\sigma^2_{\epsilon} = \alpha\sigma^2_{V}
\end{align*}
On résous ce système d'équations numériquement:
<<>>=
fun1 <- function(par,alpha,sV,sdelta)
{
sqrt(((1+alpha^2)*sV+sdelta-(1+par[1]^2)*par[2])^2
+(alpha*sV-par[1]*par[2])^2)
}
paramoptimaux1 <- round(optim(c(0.4,0.04),fun1,,0.5,0.04,0.01)$par,5)
@
On a donc que $\theta$ = \Sexpr{paramoptimaux1[1]} et $\sigma^2_{\epsilon}$ = \Sexpr{paramoptimaux1[2]}
On obtient le meme résultat en utilisant un logiciel de calcul symbolique comme Maxima.
\end{enumerate}
\end{sol}
\end{exercice}
\begin{exercice}
Trouver la valeur projetée $x_{t+2}$ et l'intervalle de confiance à 95\% pour le processus $AR(1)$ de paramètre $\phi=0.4$ et $\sigma^2_{\epsilon}=0.25$, si $x_t=1.5$.
\begin{sol}
\begin{align*}
E_t[x_{t+2}] &= \phi^2 x_t = 0.4^2*1.5 = 0.24 \\
V_t[x_{t+2}] &= (1+\phi^2)\sigma^2_{\epsilon} = (1+0.4^2)*0.25 = 0.29
\end{align*}
Intervalle de confiance:
\begin{align*}
IC &= E_t[x_{t+2}] \pm \Phi^{-1}(0.975)\sqrt{V_t[x_{t+2}]} \\
&= 0.4^2*1.5 \pm 1.96 \sqrt{(1+0.4^2)*0.25} \\
&= [-0.815492;1.295492]
\end{align*}
\end{sol}
\end{exercice}
\Closesolutionfile{solutions}
\Closesolutionfile{reponses}
%%% Local Variables:
%%% mode: latex
%%% TeX-master: "exercices_series_chrono"
%%% End:

90
serie3-exercices.Rnw Normal file
View file

@ -0,0 +1,90 @@
\chapter{Modèles de volatilité stochastique}
\label{chap:modeles-volatilite}
\Opensolutionfile{reponses}[reponses-modeles-volatilite]
\Opensolutionfile{solutions}[solutions-modeles-volatilite]
\begin{Filesave}{reponses}
\bigskip
\section*{Réponses}
\end{Filesave}
\begin{Filesave}{solutions}
\section*{Chapitre \ref{chap:modeles-volatilite}}
\addcontentsline{toc}{section}{Chapitre \protect\ref{chap:modeles-volatilite}}
\end{Filesave}
<<echo=FALSE>>=
options(width = 55)
@
\begin{exercice}
On considère un processus ARCH(2) dont le carré des résidus répond à l'équation suivante \footnote{Cet exercice est inspiré de l'exercice 8 du chapitre 3 de Enders (2004)} :
\begin{align*}
\epsilon_t^2 &= \alpha_0 + \alpha_1\epsilon_{t-1}^2 + \alpha_2\epsilon_{t-2}^2.
\end{align*}
On suppose que les résidus proviennent du modèle suivant:
\begin{align*}
y_t &= a_0 + a_1 y_{t-1} + \epsilon_t.
\end{align*}
Trouvez la variance conditionnelle et inconditionnelle de $\left\{ y_t \right\}$.
\begin{sol}
On identifie d'abord la moyenne conditionnelle de $y_t$:
\begin{align}
E_{t-1}[y_t] &= E_{t-1}[a_0 + a_1 y_{t-1} + \epsilon_t] \\
&= a_0 + a_1 y_{t-1}
\end{align}
La variance conditionnelle peut alors s'obtenir en utilisant la définition habituelle:
\begin{align*}
V_{t-1}[y_t | y_{t-1}, y_{t-2}, \ldots] &= E_{t-1}[y_t - E_{t-1}[y_t]]^{2} \\
&= E_{t-1}[(a_0 + a_1 y_{t-1} + \epsilon_t)-(a_0 + a_1 y_{t-1})]^{2} \\
&= E_{t-1}[\epsilon_t]^{2} \\
&= E_{t-1}[\alpha_0 + \alpha_1\epsilon_{t-1}^2 + \alpha_2\epsilon_{t-2}^2] \\
&= \alpha_0 + \alpha_1\epsilon_{t-1}^2 + \alpha_2\epsilon_{t-2}^2
\end{align*}
La variance inconditionnelle s'obtient en trouvant la solution particulière pour $y_t$:
\begin{align*}
•y_t &= a_0 + a_1 y_{t-1} + \epsilon_t \\
&= (1+a_1)a_0 + a_1^2 y_{t-2} + a_1 \epsilon_{t-1} + \epsilon_t \\
&= \cdots \\
&= (a+a_1+a_2+a_3+\ldots)a_0 + \epsilon_t + a_1\epsilon_{t-1} + a_2\epsilon_{t-2} + \ldots \\
&= \frac{a_0}{1-a_1} + \sum_{i=0}^{\infty} a_1^{i}\epsilon_{t-i}
\end{align*}
On évalue la variance de cette dernière expression:
\begin{align*}
•Var[y_t] &= Var[\sum_{i=0}^{\infty} a_1^{i}\epsilon_{t-i}] \\
&= \sum_{i=0}^{\infty} a_1^{2i} Var[\epsilon_{t-i}] \\
&= \frac{\sigma^2}{1-a_1^2}
\end{align*}
À partir de la définition, on a que:
\begin{align*}
E[\epsilon_t^2] &= \alpha_0 + \alpha_1 E_[\epsilon_{t-1}^2] + \alpha_2 E[\epsilon_{t-2}^2].
\end{align*}
Comme la variance inconditionnelle de $\epsilon_t$ est identique à celle de $\epsilon_{t-1}$ et $\epsilon_{t-2}$, on peut affirmer que:
\begin{align*}
E[\epsilon_t^2] &= \frac{\alpha_0}{1-\alpha_1-\alpha_2} \\
&= \sigma^2.
\end{align*}
On obtient donc que la variance inconditionnelle de $y_t$ est
\begin{align*}
•Var[y_t] &= \frac{\alpha_0}{(1-\alpha_1-\alpha_2)(1-a_1^2)}.
\end{align*}
\end{sol}
\end{exercice}
\Closesolutionfile{solutions}
\Closesolutionfile{reponses}
%%% Local Variables:
%%% mode: latex
%%% TeX-master: "exercices_series_chrono"
%%% End:

View file

@ -0,0 +1,652 @@
\section*{Chapitre \ref{chap:methodes-lissage}}
\addcontentsline{toc}{section}{Chapitre \protect\ref{chap:methodes-lissage}}
\begin{solution}{1.1}
\begin{enumerate}
\item
\begin{Schunk}
\begin{Sinput}
> library(xtable)
> library(zoo)
> library(TTR)
> Yt <- read.csv("inflation.csv",header=TRUE,sep="\t")[,2]
> Yt.ts <-ts(Yt,start=c(2008,7),deltat=1/12)
\end{Sinput}
\end{Schunk}
\begin{Schunk}
\begin{Sinput}
> xtable(Yt.ts,digits=1) ## Générer une table LaTeX
\end{Sinput}
% latex table generated in R 3.6.1 by xtable 1.8-4 package
% Thu Jan 2 17:27:11 2020
\begin{table}[ht]
\centering
\begin{tabular}{rrrrrrrrrrrrr}
\hline
& Jan & Feb & Mar & Apr & May & Jun & Jul & Aug & Sep & Oct & Nov & Dec \\
\hline
2008 & & & & & & & -0.1 & 0.5 & 0.7 & 0.9 & 1.4 & 2.3 \\
2009 & 1.5 & 0.9 & 2.2 & 0.8 & 0.2 & 0.3 & 1.0 & 0.3 & 0.8 & 0.4 & 1.6 & 2.0 \\
2010 & 3.2 & 2.3 & 1.4 & 0.6 & 0.7 & 1.1 & -0.2 & 1.4 & 0.9 & 1.4 & 1.4 & 1.9 \\
2011 & 3.1 & 2.1 & 2.7 & 1.7 & 1.7 & 0.1 & 0.9 & 1.6 & 1.6 & 2.5 & 2.4 & 2.6 \\
2012 & 2.0 & 3.2 & 2.9 & 1.4 & 1.1 & 1.3 & 1.4 & 1.4 & 1.5 & 1.7 & 2.3 & 2.4 \\
2013 & 3.0 & 2.3 & 2.3 & 1.9 & 1.7 & 0.5 & 0.9 & & & & & \\
\hline
\end{tabular}
\end{table}\end{Schunk}
On retrouve le graphique de la série $Y_t$ à la figure \ref{fig:exercice1-graph1}.
\begin{figure}[!ht]
\centering
\includegraphics[height=4in, width=4in]{exercice1-graph1.pdf}
\caption{Graphique de la série $Y_t$}
\label{fig:exercice1-graph1}
\end{figure}
\item
\begin{Schunk}
\begin{Sinput}
> xtable(Zt.ts <- diff(Yt.ts,12),digits=1)
\end{Sinput}
% latex table generated in R 3.6.1 by xtable 1.8-4 package
% Thu Jan 2 17:27:11 2020
\begin{table}[ht]
\centering
\begin{tabular}{rrrrrrrrrrrrr}
\hline
& Jan & Feb & Mar & Apr & May & Jun & Jul & Aug & Sep & Oct & Nov & Dec \\
\hline
2009 & & & & & & & 1.1 & -0.2 & 0.1 & -0.5 & 0.2 & -0.3 \\
2010 & 1.7 & 1.4 & -0.8 & -0.2 & 0.6 & 0.8 & -1.2 & 1.2 & 0.1 & 1.1 & -0.2 & -0.1 \\
2011 & -0.1 & -0.2 & 1.4 & 1.0 & 1.0 & -0.9 & 1.1 & 0.2 & 0.7 & 1.1 & 1.0 & 0.8 \\
2012 & -1.1 & 1.1 & 0.1 & -0.3 & -0.6 & 1.1 & 0.5 & -0.2 & -0.1 & -0.8 & -0.1 & -0.2 \\
2013 & 1.0 & -0.9 & -0.6 & 0.5 & 0.5 & -0.8 & -0.5 & & & & & \\
\hline
\end{tabular}
\end{table}\end{Schunk}
On retrouve le graphique de la série désaisonnalisée $Z_t$ à la figure \ref{fig:exercice1-graph2}.
\begin{figure}[!ht]
\centering
\includegraphics[height=4in, width=4in]{exercice1-graph2.pdf}
\caption{Graphique de la série désaisonnalisée $Z_t$}
\label{fig:exercice1-graph2}
\end{figure}
On élimine la composante de saisonnalité
\begin{Schunk}
\begin{Sinput}
> xtable(Yt.ts-Zt.ts,digits=1)
\end{Sinput}
% latex table generated in R 3.6.1 by xtable 1.8-4 package
% Thu Jan 2 17:27:11 2020
\begin{table}[ht]
\centering
\begin{tabular}{rrrrrrrrrrrrr}
\hline
& Jan & Feb & Mar & Apr & May & Jun & Jul & Aug & Sep & Oct & Nov & Dec \\
\hline
2009 & & & & & & & -0.1 & 0.5 & 0.7 & 0.9 & 1.4 & 2.3 \\
2010 & 1.5 & 0.9 & 2.2 & 0.8 & 0.2 & 0.3 & 1.0 & 0.3 & 0.8 & 0.4 & 1.6 & 2.0 \\
2011 & 3.2 & 2.3 & 1.4 & 0.6 & 0.7 & 1.1 & -0.2 & 1.4 & 0.9 & 1.4 & 1.4 & 1.9 \\
2012 & 3.1 & 2.1 & 2.7 & 1.7 & 1.7 & 0.1 & 0.9 & 1.6 & 1.6 & 2.5 & 2.4 & 2.6 \\
2013 & 2.0 & 3.2 & 2.9 & 1.4 & 1.1 & 1.3 & 1.4 & & & & & \\
\hline
\end{tabular}
\end{table}\end{Schunk}
On retrouve le graphique de la composante de saisonnalité $Y_t-Z_t$ à la figure \ref{fig:exercice1-graph3}.
\begin{figure}[!ht]
\centering
\includegraphics[height=4in, width=4in]{exercice1-graph3.pdf}
\caption{Graphique de la composante de saisonnalité $Y_t-Z_t$}
\label{fig:exercice1-graph3}
\end{figure}
\item
On élimime maintenant la tendance:
On utilise une moyenne mobile avec $q=1$. Comme la fonction \emph{SMA()} utilise les $2q+1$ données précédentes et que nous voulons une moyenne mobile centrée, nous devons utiliser l'opérateur de rétrodécalage $B()$ pour décaler la série.
\begin{Schunk}
\begin{Sinput}
> ## Simple Moving Average(q=1)
> xtable(mt1 <- lag(SMA(Zt.ts,n=3),1),digits=2)
\end{Sinput}
% latex table generated in R 3.6.1 by xtable 1.8-4 package
% Thu Jan 2 17:27:11 2020
\begin{table}[ht]
\centering
\begin{tabular}{rrrrrrrrrrrrr}
\hline
& Jan & Feb & Mar & Apr & May & Jun & Jul & Aug & Sep & Oct & Nov & Dec \\
\hline
2009 & & & & & & & & 0.32 & -0.22 & -0.06 & -0.21 & 0.54 \\
2010 & 0.92 & 0.77 & 0.15 & -0.12 & 0.39 & 0.07 & 0.26 & 0.05 & 0.78 & 0.32 & 0.22 & -0.16 \\
2011 & -0.13 & 0.36 & 0.74 & 1.12 & 0.36 & 0.37 & 0.10 & 0.63 & 0.63 & 0.90 & 0.93 & 0.20 \\
2012 & 0.23 & 0.03 & 0.32 & -0.24 & 0.10 & 0.37 & 0.51 & 0.09 & -0.37 & -0.34 & -0.37 & 0.25 \\
2013 & -0.02 & -0.16 & -0.33 & 0.16 & 0.10 & -0.26 & & & & & & \\
\hline
\end{tabular}
\end{table}\end{Schunk}
Moyenne mobile avec $q=5$
\begin{Schunk}
\begin{Sinput}
> ## Simple Moving Average(q=5)
> xtable(mt2 <- lag(SMA(Zt.ts,n=11),5),digits=2)
\end{Sinput}
% latex table generated in R 3.6.1 by xtable 1.8-4 package
% Thu Jan 2 17:27:11 2020
\begin{table}[ht]
\centering
\begin{tabular}{rrrrrrrrrrrrr}
\hline
& Jan & Feb & Mar & Apr & May & Jun & Jul & Aug & Sep & Oct & Nov & Dec \\
\hline
2009 & & & & & & & & & & & & 0.28 \\
2010 & 0.25 & 0.17 & 0.26 & 0.32 & 0.40 & 0.40 & 0.24 & 0.10 & 0.16 & 0.30 & 0.34 & 0.36 \\
2011 & 0.37 & 0.37 & 0.37 & 0.33 & 0.45 & 0.55 & 0.63 & 0.54 & 0.52 & 0.44 & 0.32 & 0.36 \\
2012 & 0.36 & 0.40 & 0.32 & 0.22 & 0.05 & -0.02 & 0.06 & 0.06 & -0.04 & -0.07 & 0.03 & -0.02 \\
2013 & -0.14 & -0.18 & & & & & & & & & & \\
\hline
\end{tabular}
\end{table}\end{Schunk}
Lissage exponentiel double avec $\alpha=0.75$
\begin{Schunk}
\begin{Sinput}
> ## Double Exponential Moving Average
> xtable(mt3 <- DEMA(Zt.ts,n=1,ratio=.05),digits=2)
\end{Sinput}
% latex table generated in R 3.6.1 by xtable 1.8-4 package
% Thu Jan 2 17:27:11 2020
\begin{table}[ht]
\centering
\begin{tabular}{rrrrrrrrrrrrr}
\hline
& Jan & Feb & Mar & Apr & May & Jun & Jul & Aug & Sep & Oct & Nov & Dec \\
\hline
2009 & & & & & & & 1.06 & 0.94 & 0.85 & 0.71 & 0.66 & 0.55 \\
2010 & 0.65 & 0.72 & 0.57 & 0.48 & 0.48 & 0.50 & 0.33 & 0.39 & 0.36 & 0.41 & 0.33 & 0.28 \\
2011 & 0.22 & 0.17 & 0.27 & 0.33 & 0.38 & 0.24 & 0.31 & 0.29 & 0.31 & 0.38 & 0.43 & 0.45 \\
2012 & 0.29 & 0.36 & 0.33 & 0.26 & 0.17 & 0.25 & 0.27 & 0.22 & 0.18 & 0.07 & 0.04 & 0.01 \\
2013 & 0.09 & -0.02 & -0.09 & -0.04 & 0.00 & -0.09 & -0.14 & & & & & \\
\hline
\end{tabular}
\end{table}\end{Schunk}
Régression linéaire
\begin{Schunk}
\begin{Sinput}
> t <- 0:48
> (lm1 <- lm(Zt.ts~t)) ## Modèle de régression sur une variable
\end{Sinput}
\begin{Soutput}
Call:
lm(formula = Zt.ts ~ t)
Coefficients:
(Intercept) t
0.446924 -0.009856
\end{Soutput}
\begin{Sinput}
> coeff1 <- coefficients(lm1)
\end{Sinput}
\end{Schunk}
\begin{Schunk}
\begin{Sinput}
> xtable(mt4 <- ts(coeff1[1]+t*coeff1[2],start=c(2009,7),deltat=1/12),digits=2)
\end{Sinput}
% latex table generated in R 3.6.1 by xtable 1.8-4 package
% Thu Jan 2 17:27:11 2020
\begin{table}[ht]
\centering
\begin{tabular}{rrrrrrrrrrrrr}
\hline
& Jan & Feb & Mar & Apr & May & Jun & Jul & Aug & Sep & Oct & Nov & Dec \\
\hline
2009 & & & & & & & 0.45 & 0.44 & 0.43 & 0.42 & 0.41 & 0.40 \\
2010 & 0.39 & 0.38 & 0.37 & 0.36 & 0.35 & 0.34 & 0.33 & 0.32 & 0.31 & 0.30 & 0.29 & 0.28 \\
2011 & 0.27 & 0.26 & 0.25 & 0.24 & 0.23 & 0.22 & 0.21 & 0.20 & 0.19 & 0.18 & 0.17 & 0.16 \\
2012 & 0.15 & 0.14 & 0.13 & 0.12 & 0.11 & 0.10 & 0.09 & 0.08 & 0.07 & 0.06 & 0.05 & 0.04 \\
2013 & 0.03 & 0.02 & 0.01 & 0.00 & -0.01 & -0.02 & -0.03 & & & & & \\
\hline
\end{tabular}
\end{table}\end{Schunk}
On retrouve le graphique de la tendance $m_t$ à la figure \ref{fig:exercice1-graph4}.
\begin{figure}[!ht]
\centering
\includegraphics[height=4in, width=4in]{exercice1-graph4.pdf}
\caption{Graphique de la tendance $m_t$}
\label{fig:exercice1-graph4}
\end{figure}
\item
\begin{Schunk}
\begin{Sinput}
> projection <- coeff1[1]+53*coeff1[2]
> saisonnalite <- mean((Yt.ts-Zt.ts)[6+12*0:3])
> (taux.inf.dec.2013 <- (projection+saisonnalite))
\end{Sinput}
\begin{Soutput}
(Intercept)
2.137743
\end{Soutput}
\end{Schunk}
Le taux d'inflation prejeté en décembre 2013 est 2.14\%
\item
\begin{Schunk}
\begin{Sinput}
> depense.dec.2008 <- 674
> depense.dec.2013 <- 674*(1+taux.inf.dec.2013/100)
\end{Sinput}
\end{Schunk}
Le montant projeté des achats de cadeaux en décembre 2013 est 688.41 \$
\end{enumerate}
\end{solution}
\begin{solution}{1.2}
On remarque d'abord que $q=2$.
On peut ensuite poser les équations suivantes:
\begin{align}
\label{eq:1}
4+3+a+b+2 &= 24\\
b+2+4+6+c &= 26\\
c+0+2+8+3 &= 19
\end{align}
En résolvant, on obtient la solution.\\
\textbf{Solution:}\\
\begin{tabular}{|l|l|l|}
\hline
\multicolumn{1}{|l|}{Mois} & \multicolumn{1}{l|}{Incendies} & \multicolumn{1}{l|}{Moyenne Mobile} \\ \hline
1 & 4 & \multicolumn{1}{l|}{-} \\ \hline
2 & 3 & \multicolumn{1}{l|}{-} \\ \hline
3 & \textbf{7} & 4,8 \\ \hline
4 & \textbf{8} & 4,8 \\ \hline
5 & 2 & 5,4 \\ \hline
6 & 4 & 5,2 \\ \hline
7 & 6 & 3,6 \\ \hline
8 & \textbf{6} & 3,6 \\ \hline
9 & 0 & 4,4 \\ \hline
10 & 2 & 3,8 \\ \hline
11 & 8 & \multicolumn{1}{l|}{-} \\ \hline
12 & 3 & \multicolumn{1}{l|}{-} \\ \hline
\end{tabular}
\end{solution}
\begin{solution}{1.3}
\begin{Schunk}
\begin{Sinput}
> rf <- 0.0175
> rB <- rf+0.02
> S0 <- 10.46
> ST <- 8.73
> K <- S0*exp(rf*84/365)
> bbry <- read.csv("blackberry.csv",header=TRUE,sep=";")
> bbry.sel <- bbry[as.POSIXlt(bbry$Date)$wday==5,][1+3:12*4,]$Close
> l.bbry.sel <- log(bbry.sel)
> (diff.l.bbry.sel <- diff(l.bbry.sel))
\end{Sinput}
\begin{Soutput}
[1] 0.288637639 0.112996047 -0.061344651 -0.103095509
[5] -0.017497594 -0.086523113 0.005008358 -0.340978628
[9] -0.090637274
\end{Soutput}
\begin{Sinput}
> (mu.diff.l.bbry.sel <- mean(diff.l.bbry.sel))
\end{Sinput}
\begin{Soutput}
[1] -0.03260386
\end{Soutput}
\begin{Sinput}
> (sigma.diff.l.bbry.sel <- sd(diff.l.bbry.sel))
\end{Sinput}
\begin{Soutput}
[1] 0.170735
\end{Soutput}
\begin{Sinput}
> (prix.arbre <- S0*(ud <- exp(3*(mu.diff.l.bbry.sel+c(1,-1)*
+ sigma.diff.l.bbry.sel/(2*sqrt(3))))))
\end{Sinput}
\begin{Soutput}
[1] 10.996838 8.181586
\end{Soutput}
\begin{Sinput}
> (p.rn <- (exp(rf*84/365)-ud[2])/(ud[1]-ud[2]))
\end{Sinput}
\begin{Soutput}
[1] 0.8243048
\end{Soutput}
\begin{Sinput}
> q.rn <- 1-p.rn
> (P0 <- sum(exp(-rf*84/365)*(c(p.rn,q.rn)*pmax(K-prix.arbre,0))))
\end{Sinput}
\begin{Soutput}
[1] 0.406084
\end{Soutput}
\begin{Sinput}
> (BT <- P0*exp(rB*84/365))
\end{Sinput}
\begin{Soutput}
[1] 0.4096037
\end{Soutput}
\begin{Sinput}
> (K-ST)-BT
\end{Sinput}
\begin{Soutput}
[1] 1.362608
\end{Soutput}
\end{Schunk}
La valeur du paramètre $\mu$ de rendement moyen est -0.0326.
La valeur du paramètre $\sigma$ de volatilité est 0.1707.
La valeur des prix de l'arbre binomial sont 10.9968 et 8.1816.
La valeur de la probabilité neutre au risque d'une hausse est 0.8243.
La valeur de l'option est 0.4061.
Le profit, qui correspont à la différence entre la réclamation contingente de l'option et le coût d'acquisition, est de 1.3626.
\end{solution}
\begin{solution}{1.4}
On calcule d'abord les deux séries lissées \\
\begin{tabular}{|l|r|r|}
\hline
\multicolumn{1}{|c|}{$\mathcal{A}$} & $\alpha=0,4$ & $\alpha=0,7$ \\ \hline
1,2 & 1,2000 & 1,2000 \\ \hline
1,5 & 1,3200 & 1,4100 \\ \hline
1,4 & 1,3520 & 1,4030 \\ \hline
2,1 & 1,6512 & 1,8909 \\ \hline
1,8 & 1,7107 & 1,8273 \\ \hline
1,9 & 1,7864 & 1,8782 \\ \hline
2,2 & 1,9519 & 2,1035 \\ \hline
2,4 & 2,1311 & 2,3110 \\ \hline
2,0 & 2,0787 & 2,0933 \\ \hline
1,9 & 2,0072 & 1,9580 \\ \hline
\end{tabular} \\
On évalue ensuite l'erreur quadratique pour chaque terme \\
\begin{tabular}{|l|r|r|}
\hline
\multicolumn{1}{|c|}{$\mathcal{A}$} & \multicolumn{1}{l|}{$SE(\alpha=0,4)$} & \multicolumn{1}{l|}{$SE(\alpha=0,7)$} \\ \hline
1,2 & \multicolumn{1}{l|}{} & \multicolumn{1}{l|}{} \\ \hline
1,5 & 0,0324 & 0,0081 \\ \hline
1,4 & 0,0023 & 0,0000 \\ \hline
2,1 & 0,2014 & 0,0437 \\ \hline
1,8 & 0,0080 & 0,0007 \\ \hline
1,9 & 0,0129 & 0,0005 \\ \hline
2,2 & 0,0616 & 0,0093 \\ \hline
2,4 & 0,0723 & 0,0079 \\ \hline
2,0 & 0,0062 & 0,0087 \\ \hline
1,9 & 0,0115 & 0,0034 \\ \hline
\end{tabular} \\
On obtient enfin l'erreur quadratique moyenne\\
\begin{tabular}{|l|l|l|}
\hline
& $\alpha=0,4$ & $\alpha=0,7$ \\ \hline
MSE & \multicolumn{1}{r|}{0,0454} & \multicolumn{1}{r|}{0,0092} \\ \hline
\end{tabular} \\
Les calculs effectués se trouvent dans le fichier \url{Lissage.Exponentiel.I.ods} \footnote{Ce fichier est au format OpenDocument et s'ouvre avec la plupart des suites bureautiques}.
Avec R, on obtient les résultats suivants en utilisant la fonction de lissage exponentiel \emph{EMA()}.
\begin{Schunk}
\begin{Sinput}
> A <- c(1.2, 1.5, 1.4, 2.1, 1.8, 1.9, 2.2, 2.4, 2.0, 1.9)
> n.A <- length(A)
> A.EMA.4 <- EMA(A,n=1,ratio=0.4)
> A.EMA.7 <- EMA(A,n=1,ratio=0.7)
> A.SE <- (A-cbind(A.EMA.4,A.EMA.7))^2
> cbind(A,A.EMA.4,A.EMA.7,A.SE)
\end{Sinput}
\begin{Soutput}
A A.EMA.4 A.EMA.7 A.EMA.4 A.EMA.7
[1,] 1.2 1.200000 1.200000 0.000000000 0.0000000000
[2,] 1.5 1.320000 1.410000 0.032400000 0.0081000000
[3,] 1.4 1.352000 1.403000 0.002304000 0.0000090000
[4,] 2.1 1.651200 1.890900 0.201421440 0.0437228100
[5,] 1.8 1.710720 1.827270 0.007970918 0.0007436529
[6,] 1.9 1.786432 1.878181 0.012897691 0.0004760688
[7,] 2.2 1.951859 2.103454 0.061573857 0.0093210722
[8,] 2.4 2.131116 2.311036 0.072298864 0.0079145417
[9,] 2.0 2.078669 2.093311 0.006188861 0.0087069216
[10,] 1.9 2.007202 1.957993 0.011492180 0.0033632189
\end{Soutput}
\begin{Sinput}
> (A.MSE <- colMeans(A.SE)*(n.A/(n.A-1)))
\end{Sinput}
\begin{Soutput}
A.EMA.4 A.EMA.7
0.04539420 0.00915081
\end{Soutput}
\end{Schunk}
La valeur $\alpha=0.7$ produit une erreur quadratique moyenne inférieure.
\end{solution}
\begin{solution}{1.5}
Une solution assez simple est d'utiliser le solveur intégré au logiciel tableau que vous utilisez et d'optimiser la valeur de la cellule contenant $\alpha$ avec comme critère de minimisation la cellule contenant l'erreur quadratique moyenne (MSE).
On peut aussi construire une fonction d'optimisation dans R qui réplique le comportement du chiffrier que nous avons construit dans le logiciel tableur.
\begin{Schunk}
\begin{Sinput}
> funOptAlphaDEMA <- function(alpha,data)
+ {
+ data.n <- length(data)
+ data.DEMA <- DEMA(A,n=1,ratio=alpha)
+ data.SE <- (data-data.DEMA)^2
+ data.MSE <- mean(data.SE)*data.n/(data.n-1)
+ print(c(data.MSE,alpha))
+ data.MSE
+ }
> optimize(funOptAlphaDEMA,c(0.4,0.7),A)
\end{Sinput}
\begin{Soutput}
[1] 0.006416607 0.514589803
[1] 0.003674206 0.585410197
[1] 0.002489337 0.629179607
[1] 0.001912478 0.656230590
[1] 0.001607733 0.672949017
[1] 0.001437559 0.683281573
[1] 0.001338969 0.689667444
[1] 0.00128047 0.69361413
[1] 0.001245226 0.696053315
[1] 0.001223788 0.697560814
[1] 0.001210668 0.698492500
[1] 0.001202609 0.699068314
[1] 0.001197648 0.699424186
[1] 0.001194588 0.699644128
[1] 0.0011927 0.6997801
[1] 0.001191534 0.699864069
[1] 0.001190814 0.699915990
[1] 0.00119025 0.69995669
[1] 0.00119025 0.69995669
$minimum
[1] 0.6999567
$objective
[1] 0.00119025
\end{Soutput}
\end{Schunk}
Tout comme pour le lissage exponentiel effectué à la question précédente, la valeur $\alpha=0.7$ produit une erreur quadratique moyenne inférieure.
\end{solution}
\begin{solution}{1.6}
On importe d'abord l'ensemble de données
\begin{Schunk}
\begin{Sinput}
> BoA <- ts(read.csv("BoA.csv",header=TRUE,sep="\t"))
\end{Sinput}
\end{Schunk}
\begin{enumerate}
\item
On trace ensuite le corrélogramme (figure \ref{fig:exercice1.6-graph1})
La fonction \emph{acf} nous permet d'afficher un corrélogramme
\begin{figure}[!ht]
\centering
\includegraphics[height=4in, width=4in]{exercice1-6-graph1}
\caption{Corrélogramme de la série BoA}
\label{fig:exercice1.6-graph1}
\end{figure}
La fonction d'autocorrélation empirique $\hat{\rho}$ prend les valeurs
suivantes:
\begin{Schunk}
\begin{Sinput}
> (BoA.acf <- acf(BoA[,2],lag.max=19))
\end{Sinput}
\begin{Soutput}
Autocorrelations of series BoA[, 2], by lag
0 1 2 3 4 5 6
1.000 0.817 0.587 0.325 0.105 -0.059 -0.161
7 8 9 10 11 12 13
-0.207 -0.272 -0.291 -0.325 -0.293 -0.249 -0.203
14 15 16 17 18 19
-0.106 -0.078 -0.057 -0.038 0.003 0.002
\end{Soutput}
\begin{Sinput}
> dummy <- dev.off()
\end{Sinput}
\end{Schunk}
En utilisant la méthode vue dans le cours, on construit un intervalle de
confiance au niveau $1-\alpha=0.9$ à partir de la distribution normale. La
valeur de $n$ est 20.
\begin{Schunk}
\begin{Sinput}
> (BoA.acf.IC <- round(c(1/sqrt(20)*qnorm(0.05),-1/sqrt(20)*qnorm(0.05)),4))
\end{Sinput}
\begin{Soutput}
[1] -0.3678 0.3678
\end{Soutput}
\end{Schunk}
\begin{align*}
IC &= \frac{1}{\sqrt{n}}\left[-z_{\alpha / 2},z_{\alpha / 2} \right] \\
&= \frac{1}{\sqrt{20}}\left[-z_{0.05},z_{0.05} \right] \\
&= \left[ -0.3678,0.3678 \right]
\end{align*}
\begin{Schunk}
\begin{Sinput}
> (BoA.nbacfplus <- sum(BoA.acf$acf[-1]<BoA.acf.IC[1]) +
+ sum(BoA.acf$acf[-1]>BoA.acf.IC[2]))
\end{Sinput}
\begin{Soutput}
[1] 2
\end{Soutput}
\end{Schunk}
Comme 2 valeurs sur 20, soit 10\% de celles-ci, sont à l'extérieur de
l'intervalle de confiance, alors on ne peut rejeter l'hypothèse selon laquelle la
série est stationnaire lorsqu'on se base sur le test du corrélogramme.
\item
Ici, on n'a qu'à tracer la série et compter les changements de direction
(figure \ref{fig:exercice1.6-graph2})
\begin{figure}[!ht]
\centering
\includegraphics[height=4in, width=4in]{exercice1-6-graph2}
\caption{Corrélogramme de la série BoA}
\label{fig:exercice1.6-graph2}
\end{figure}
On en dénombre 9.
\begin{Schunk}
\begin{Sinput}
> BoA.chdir <- abs((9-(2/3)*18)/sqrt((16*20-29)/90))
> BoA.chdir > qnorm(0.95)
\end{Sinput}
\begin{Soutput}
[1] TRUE
\end{Soutput}
\end{Schunk}
On évalue la statistique de test, qui prend la valeur 1.6684. Comme cette valeur est supérieure au seuil de 1.6449, on rejette l'hypothèse de stationnarité avec le test du changement de direction.
\item
Il existe deux tests de type Portmanteau. Celui que vous avez vu en classe est le test de Box-Pierce où h commence à 1.
\begin{Schunk}
\begin{Sinput}
> Box.test(BoA[,2],lag=19,type="Box-Pierce")
\end{Sinput}
\begin{Soutput}
Box-Pierce test
data: BoA[, 2]
X-squared = 33.518, df = 19, p-value = 0.02093
\end{Soutput}
\begin{Sinput}
> qchisq(0.9,19)
\end{Sinput}
\begin{Soutput}
[1] 27.20357
\end{Soutput}
\end{Schunk}
On rejette l'hypothèse de stationnarité car la valeur de $Q^{*}=33.5182$ est supérieure au quantile $\chi^2_{0.1}(19) = 27.20357$
\item
Les tests sont indépendants, différents entre eux et ne sont pas équivalents
car leurs statistiques ne suivent pas la même distribution asymptotique.
\item
\begin{Schunk}
\begin{Sinput}
> round(diff(log(BoA[,2])),4)
\end{Sinput}
\begin{Soutput}
Time Series:
Start = 2
End = 20
Frequency = 1
[1] 0.0000 0.0381 0.0035 -0.0090 -0.0176 0.0314
[7] -0.0104 0.0021 0.0267 -0.0074 0.0014 -0.0681
[13] -0.0537 -0.0154 -0.0133 0.0295 0.0235 0.0200
[19] -0.0313
\end{Soutput}
\end{Schunk}
\item
\begin{Schunk}
\begin{Sinput}
> (BoA.hist.var <- zoo::na.trim(apply(cbind(BoA[,2],
+ lag(BoA[,2],1),
+ lag(BoA[,2],2),
+ lag(BoA[,2],3),
+ lag(BoA[,2],4)),
+ 1,
+ var)))
\end{Sinput}
\begin{Soutput}
[1] 0.08642 0.06185 0.03017 0.02963 0.02753 0.06795
[7] 0.03257 0.03617 0.18785 0.61597 0.79813 0.71857
[13] 0.17257 0.06697 0.15075 0.12818
\end{Soutput}
\begin{Sinput}
> Box.test(BoA.hist.var,lag=15,type="Box-Pierce")
\end{Sinput}
\begin{Soutput}
Box-Pierce test
data: BoA.hist.var
X-squared = 13.708, df = 15, p-value = 0.5478
\end{Soutput}
\begin{Sinput}
> qchisq(0.9,15)
\end{Sinput}
\begin{Soutput}
[1] 22.30713
\end{Soutput}
\end{Schunk}
On remarque que la série des volatilités historiques avec $q=2$ est stationnaire bien que la volatilité ne soit pas constante.
\end{enumerate}
\end{solution}
\begin{solution}{1.7}
La variance du $t^e$ terme est équivalente à la somme de la variance des $t$ premiers termes d'erreurs. La différence entre les variances des $5^e$ terme et du $7^e$ terme est donc égale à la somme:
\begin{align*}
\label{eq:2}
V\left[\epsilon_6\right]+V\left[\epsilon_7\right] &= 0.1(6^2+7^2) \\
& = 8.5
\end{align*}
\end{solution}

View file

@ -0,0 +1,420 @@
\section*{Chapitre \ref{chap:modeles-classiques}}
\addcontentsline{toc}{section}{Chapitre \protect\ref{chap:modeles-classiques}}
\begin{solution}{2.1}
On obtient les racines de l'équation caractéristique en utilisant la formule quadratique habituelle
\begin{align*}
\frac{\phi_1\pm\sqrt{\phi_1^2+4\phi_2}}{-2\phi_2}
\end{align*}
On considère les inverses des deux racines, $A_1$ et $A_2$.
\begin{align*}
A_1 &= \frac{2\phi_2}{-\phi_1-\sqrt{\phi_1^2+4\phi_2}} \\
&= \frac{2\phi_2}{-\phi_1-\sqrt{\phi_1^2+4\phi_2}} \left[\frac{-\phi_1+\sqrt{\phi_1^2+4\phi_2}}{-\phi_1+\sqrt{\phi_1^2+4\phi_2}} \right] \\
&= \frac{2\phi_2(-\phi_1+\sqrt{\phi_1^2+4\phi_2})}{\phi_1^2-(\phi_1^2+4\phi_2)}\\
&= \frac{\phi_1-\sqrt{\phi_1^2+4\phi_2}}{2}\\
A_2 &= \frac{\phi_1+\sqrt{\phi_1^2+4\phi_2}}{2}
\end{align*}
Il y a 2 situations possibles: soit les racines sont réelles ($\phi_1^2+4\phi_2>0$) ou elles sont complexes ($\phi_1^2+4\phi_2<0$).
\begin{itemize}
\item \textbf{Racines réelles:}
Comme les racines doivent être plus grandes que 1, alors nécessairement leurs inverses $|A_1|<1$ et $|A_2|<1$. Nous avons donc:
\begin{align*}
-1 &< \frac{\phi_1-\sqrt{\phi_1^2+4\phi_2}}{2} < \frac{\phi_1+\sqrt{\phi_1^2+4\phi_2}}{2} < 1 \\
\Leftrightarrow -2 &< \phi_1-\sqrt{\phi_1^2+4\phi_2} < \phi_1+\sqrt{\phi_1^2+4\phi_2} < 2
\end{align*}
En observant la première inégalité, on a:
\begin{align*}
-2 < \phi_1-\sqrt{\phi_1^2+4\phi_2}
&\Leftrightarrow \sqrt{\phi_1^2+4\phi_2}<\phi_1+2 \\
&\Leftrightarrow \phi_1^2+4\phi_2 < \phi_1^2+4\phi_1+4 \\
&\Leftrightarrow \phi_2 < \phi_1 + 1 \\
&\Leftrightarrow \phi_2 - \phi_1 < 1
\end{align*}
Ce qui correspond à la seconde condition. En considérant la seconde inégalité, on obtient, de la même façon, la première inégalité:
\begin{align*}
\phi_1+\sqrt{\phi_1^2+4\phi_2} &< 2 \\
&\Leftrightarrow \phi_1 + \phi_2 < 1
\end{align*}
Ces deux conditions réunies avec un discriminant positif forment la région de stationnarité pour des racines réelles.
\item \textbf{Racines complexes:}
On considère la situation où $\phi_1^2+4\phi_2<0$. Ici, on aura des conjugués complexes et $|A_1| = |A_2| <1$ seulement si $|A_1|^2<1$.
\begin{align*}
|A_1|^2 &=\frac{\phi_1^2+(-\phi_1^2-4\phi_2}{4}=-\phi^2 \\
&\Leftrightarrow \phi_2>-1 \\
&\Leftrightarrow |\phi_2|<1
\end{align*}
Ce résultat réuni avec un discriminant négatif forment la région de stationnarité pour des racines complexes.
\end{itemize}
\end{solution}
\begin{solution}{2.2}
\begin{enumerate}
\item On obtient ce résultat par récurrence. Par exemple, pour $k=2$, on a :
\begin{align*}
(1-B)^2 (a_0+a_1t+a_2t^2) &= (1-B) ((a_0+a_1t+a_2t^2)\\ &\quad- (a_0+a_1(t-1)+a_2(t-1)^2)) \\
&= (1-B) (a_1+a_2(2t+1)) \\
&= (a_1+a_2(2t+1)) - (a_1+a_2(2(t-1)+1)) \\
&= 2a_2
\end{align*}
En général, on obtient:
\begin{align*}
(1-B)^k (a_0+a_1t+a_2t^2+\ldots+a_kt^k) &= (1-B)^{k-1} ((a_0+a_1t+a_2t^2+\ldots+a_kt^k)\\ &\quad- (a_0+a_1(t-1)+a_2(t-1)^2+\ldots+a_k(t-1)^k)) \\
&= (1-B)^{k-1} (a_1 + 2a_2t+\ldots+a_k(t^k-(t-1)^k))
\end{align*}
On remarque qu'à chaque itération, le premier terme de la série disparait. Ainsi, après $k$ itérations, il ne restera que le terme en $a_k$ avec son coefficient, qui correspont à $k!$. On obtient ainsi la solution générale.
\item Une série est dite stationnaire lorsque chaque terme est un terme d'erreur dont la distribution est constante au fil du temps. Ainsi, la distribution de la différence de deux termes consécutifs de la série sera aussi constante au fil du temps. Par exemple:
\begin{align*}
\epsilon_t \sim N(0,\sigma^2) \\
\epsilon_t - \epsilon_{t-1} \sim N(0,2\sigma^2) \\
\end{align*}
\end{enumerate}
\end{solution}
\begin{solution}{2.3}
\begin{enumerate}
\item Un processus AR(1) est défini par $y_t = \phi_1y_{t-1} + \epsilon_t$. En développant le terme $y_{t-1}$, on obtient $y_t = \phi_1^2y_{t-2}+\phi_1\epsilon_{t-1}+\epsilon_t$. De manière récursive, on obtient $y_t = \phi_1^{t}\epsilon_0 + \phi_1^{t-1}\epsilon_1 + \ldots + \phi_1\epsilon_{t-1} + \epsilon_t$. ainsi, en faisant tendre $t\to\infty$, on obtient une représentation MA($\infty$).
\item Un processus MA(1) est défini par $y_t = \epsilon_t - \theta_1\epsilon_{t-1}$. On cherche à substituer le terme $\epsilon_{t-1}$. On développe le terme précédent de la série: $y_{t-1} = \epsilon_{t-1} - \theta_1\epsilon_{t-2}$ et on substitue dans la première expression pour obtenir $y_t = \epsilon_t - \theta_1y_{t-1} - \theta_1^2\epsilon_{t-2}$. De manière récursive, on obtient $y_t = -\theta_1^ty_0-\theta_1^{t-1}y_1-\ldots-\theta_1y_{t-1}+\epsilon_{t}$. Lorsque $t\to\infty$, on obtient une représentation AR($\infty$).
\end{enumerate}
\end{solution}
\begin{solution}{2.4}
On utilise la formule $y_t = \phi_1y_{t-1} + \epsilon_t$.
\begin{center}
\begin{tabular}{|r|r|r|}
\hline
\multicolumn{1}{|l|}{} & \multicolumn{ 2}{c|}{$\phi$} \\ \hline
\multicolumn{1}{|l|}{$N(0,1)$} & -0,5 & 0,5 \\ \hline
-1,21 & -1,2100 & -1,2100 \\ \hline
0,28 & 0,8850 & -0,3250 \\ \hline
1,08 & 0,6375 & 0,9175 \\ \hline
-2,35 & -2,6688 & -1,8913 \\ \hline
0,43 & 1,7644 & -0,5156 \\ \hline
0,51 & -0,3722 & 0,2522 \\ \hline
-0,57 & -0,3839 & -0,4439 \\ \hline
-0,55 & -0,3580 & -0,7720 \\ \hline
-0,56 & -0,3810 & -0,9460 \\ \hline
-0,89 & -0,6995 & -1,3630 \\ \hline
\end{tabular}
\end{center}
Les séries avec une corrélation négative ont tendance à aller dans la direction contraire des termes précédents alors que celles avec une corrélation positive ont tendance à aller dans la même direction que les termes précédents.
Le tableur \url{constructionserieAR.ods} contient les calculs effectués.
\end{solution}
\begin{solution}{2.5}
Il suffit de calculer la fonction d'autorégression pour chaque processus MA(2).
Ensuite, on peut évaluer la fonction d'autocorrélation et comparer le résultat obtenu.
\begin{enumerate}
\item Premier processus avec:
\begin{align*}
\theta_1 = \theta_2 = \frac{1}{4}
\end{align*}
Fonction d'autocovariance
\begin{align*}
\phi_0^{(1)} &= V[Y_t] \\
&= V[e_t]+\frac{1}{16}V[e_{t-1}]+\frac{1}{16}V[e_{t-2}] \\
&= (1+\frac{1}{8})\theta^2_e \\
&= \frac{9}{8} \sigma^2_e \\
\end{align*}
\begin{align*}
\phi_1^{(1)} &= Cov(Y_t,Y_{t-1}) \\
&= Cov(e_t - \frac{1}{4}e_{t-1} - \frac{1}{4}e_{t-2}, e_{t-1} - \frac{1}{4}e_{t-2} - \frac{1}{4}e_{t-3} )\\
&= Cov(-\frac{1}{4}e_{t-1},e_{t-1}) + Cov(-\frac{1}{4}e_{t-2},-\frac{1}{4}e_{t-2}) \\
&= (-\frac{1}{4}+(-\frac{1}{4})(-\frac{1}{4})) \sigma^2_e \\
&= -\frac{3}{16} \sigma^2_e \\
\end{align*}
\begin{align*}
\phi_2^{(1)} &= Cov(Y_t,Y_{t-2}) \\
&= Cov(e_t - \frac{1}{4}e_{t-1} - \frac{1}{4}e_{t-2}, e_{t-2} - \frac{1}{4}e_{t-3} - \frac{1}{4}e_{t-4} )\\
&= Cov(-\frac{1}{4}e_{t-2},e_{t-2}) \\
&= -\frac{1}{4} \sigma^2_e \\
\end{align*}
\begin{align*}
\phi_k^{(1)} &= 0, \qquad \forall k \geq 3
\end{align*}
Fonction d'autocorrélation
\begin{align*}
\rho_1^{(1)} &= \frac{\phi_1^{(1)}}{\phi_0^{(1)}} \\
&= \frac{\frac{-3}{16}\sigma^2_e}{\frac{9}{8}\sigma^2_e} \\
&= \frac{-1}{6} \\
\end{align*}
\begin{align*}
\rho_2^{(1)} &= \frac{\phi_2^{(1)}}{\phi_0^{(1)}} \\
&= \frac{\frac{-1}{4}\sigma^2_e}{\frac{9}{8}\sigma^2_e} \\
&= \frac{-2}{9} \\
\end{align*}
\item Second processus avec:
\begin{align*}
\theta_1 = -1 \quad \theta_2 = 4
\end{align*}
Fonction d'autocovariance
\begin{align*}
\phi_0^{(2)} &= V[Y_t] \\
&= V[e_t]+V[e_{t-1}]+16V[e_{t-2}] \\
&= 18 \sigma^2_e
\end{align*}
\begin{align*}
\phi_1^{(2)} &= Cov(Y_t,Y_{t-1}) \\
&= Cov(e_t + e_{t-1} - 4e_{t-2}, e_{t-1} + e_{t-2} - 4e_{t-3}) \\
&= Cov(e_{t-1},e_{t-1}) + Cov(-4e_{t-2},e_{t-2}) \\
&= (1 + (-1)(4))\sigma^2_e \\
&= -3\sigma^2_e \\
\end{align*}
\begin{align*}
\phi_2^{(2)} &= Cov(Y_t,Y_{t-2}) \\
&= Cov(e_t + e_{t-1} - 4e_{t-2}, e_{t-2} + e_{t-3} - 4e_{t-4} )\\
&= Cov(- 4e_{t-2},e_{t-2}) \\
&= -4\sigma^2_e \\
\end{align*}
Fonction d'autocorrélation
\begin{align*}
\rho_1^{(2)} &= \frac{\phi_1^{(2)}}{\phi_0^{(2)}} \\
&= \frac{-3\sigma^2_e}{18\sigma^2_e} \\
&= \frac{-1}{6} \\
\end{align*}
\begin{align*}
\rho_2^{(2)} &= \frac{\phi_2^{(2)}}{\phi_0^{(2)}} \\
&= \frac{-4\sigma^2_e}{18\sigma^2_e} \\
&= \frac{-2}{9} \\
\end{align*}
\end{enumerate}
On remarque clairement que $\rho_1^{(1)} = \rho_1^{(2)}$ et $\rho_2^{(1)} = \rho_2^{(2)}$.
La fonction d'autocovariance vaut toujours 1 pour $\rho_1$ et vaut 0 ailleurs.
\end{solution}
\begin{solution}{2.6}
\begin{enumerate}
\item
Les deux équations de Yule-Walker pour le modèle AR(2) sont les suivantes:
\begin{align*}
\rho_1 = \phi_1 + \rho_1 \phi_2 \\
\rho_2 = \rho_1\phi_1 + \phi_2
\end{align*}
En utilisant l'estimateur de la fonction d'autocovariance $\hat{\rho}$, on obtient alors:
\begin{align*}
\hat{\phi_1} = \frac{\hat{\rho_1}(1-\hat{\rho_2})}{1-\hat{\rho_1}^2} \\
\hat{\phi_2} = \frac{\hat{\rho_2} - \hat{\rho_1}^2}{1-\hat{\rho_1}^2}
\end{align*}
\item
\begin{Schunk}
\begin{Sinput}
> set.seed(123)
> (serie <- arima.sim(n = 10, list(ar = c(0.5,-0.25))))
\end{Sinput}
\begin{Soutput}
Time Series:
Start = 1
End = 10
Frequency = 1
[1] 1.1617660 0.6981185 0.1693004 -0.6457205
[5] 1.4217278 1.3701445 -1.6369769 -0.4596686
[9] -0.2933815 -1.0995973
\end{Soutput}
\begin{Sinput}
> acf.serie <- acf(serie,type="correlation",plot=FALSE)$acf[2:3]
> phi1 <- acf.serie[1]*(1-acf.serie[2]) / (1-acf.serie[1]^2)
> phi2 <- (acf.serie[2] - acf.serie[1]^2) / (1-acf.serie[1]^2)
\end{Sinput}
\end{Schunk}
On obtient $\hat{\rho_1} = 0.07455$ et $\hat{\rho_2} = -0.28041$.
Ce qui nous donne les paramètres du modèle AR(2) suivants:
$\hat{\phi_1} = 0.09599$ et
$\hat{\phi_2} = -0.28757$.
\end{enumerate}
\end{solution}
\begin{solution}{2.7}
\begin{enumerate}
\item
On représente le processus ARMA(2,1) sous la forme suivante:
\begin{align*}
y_t = \mu + \phi_1 y_{t-1} + \phi_2 y_{t-2} + \epsilon_t - \theta\epsilon_{t-1}
\end{align*}
En utilisant l'opérateur de rétrodécalage $B$, on peut exprimer cette équation sous la forme suivante:
\begin{align*}
(1-\theta B)\epsilon_t = y_t - \mu - \phi_1 y_{t-1} - \phi_2 y_{t-2}
\end{align*}
On divise ensuite de chaque côté par $(1-\theta B)$, pour obtenir:
\begin{align*}
\epsilon_t = \frac{1}{(1-\theta B)} \left(y_t - \mu - \phi_1 y_{t-1} - \phi_2 y_{t-2}\right)
\end{align*}
L'hypothèse de stationnarité nous permet de poser que $| \theta | < 1$, ce qui nous permet d'utiliser la série géométrique définie comme suit:
\begin{align*}
\frac{1}{1-\theta B} = \sum_{i=0}^{\infty} \theta^i B^i
\end{align*}
On obtient donc que
\begin{align*}
\epsilon_t = \sum_{i=0}^{\infty} \theta^i B^i \left(y_t - \mu - \phi_1 y_{t-1} - \phi_2 y_{t-2}\right)
\end{align*}
En appliquant l'opérateur de rétrodécalage à la parenthèse, on obtient la solution:
\begin{align*}
\epsilon_t = \sum_{i=0}^{\infty} \theta^i \left(y_{t-i} - \mu - \phi_1 y_{t-i-1} - \phi_2 y_{t-i-2} \right)
\end{align*}
\item
On exprime l'équation précédente en fonction de $y_t$:
\begin{align*}
y_t &= \phi_1 y_{t-1} + \phi_2 y_{t-2} - \sum_{i=1}^{\infty} \theta^i \left(y_{t-i} - \phi_1 y_{t-i-1} - \phi_2 y_{t-i-2} \right) + \epsilon_t + \frac{\mu}{1-\theta} \\
&= \phi_1 y_{t-1} + \phi_2 y_{t-2} - \left[\theta\left(y_{t-1}-\phi_1 y_{t-2} - \phi_2 y_{t-3} \right) + \sum_{i=2}^{\infty} \theta^i \left(y_{t-i} - \phi_1 y_{t-i-1} - \phi_2 y_{t-i-2} \right)\right] + \epsilon_t + \frac{\mu}{1-\theta} \\
&= (\phi_1 - \theta) y_{t-1} - \sum_{i=2}^{\infty} \left(\theta_i+\phi_1\theta^{i-1} - \phi_2\theta_{i-2}\right) y_{t-i} + \epsilon_t + \frac{\mu}{1-\theta}
\end{align*}
Ce qui correspont à la forme autorégressive AR($\infty$) suivante:
\begin{align*}
y_t = c + \sum_{i=1}^{\infty} \pi_i y_{t-i} + \epsilon_t
\end{align*}
Avec les paramètres
\begin{align*}
c &= \frac{\mu}{1-\theta} \\
\pi_1 &= (\phi_1 - \theta) \\
\pi_i &= -\left(\theta_i+\phi_1\theta^{i-1} - \phi_2\theta_{i-2}\right),\quad i=2,3,\ldots
\end{align*}
\end{enumerate}
\end{solution}
\begin{solution}{2.8}
\begin{enumerate}
\item C'est un modèle AR(2) de paramètres $\phi_1=1.5$ et $\phi_2=-0.5$.
\item L'équation caractéristique prend la forme suivante:
\begin{align*}
y_t - 1.5 y_{t-1} + 0.5y_{t-2} = 0
\end{align*}
On pose la solution générale $y_t=A\alpha^t$:
\begin{align*}
A\alpha^t - 1.5 A\alpha^{t-1} + 0.5A\alpha^{t-2} = 0
\end{align*}
On divise ensuite par $A\alpha^{t-2}$:
\begin{align*}
0 &= \alpha^2 - 1.5 \alpha + 0.5\\
\alpha &= \frac{1.5 \pm \sqrt{1.5^2-4*1*0.5}}{2} \\
&= \left\{0.5 ; 1 \right\}
\end{align*}
\item
\begin{align*}
1 - 1.5B + 0.5B^2 &= 0 \\
(1-B)(1-0.5B) &= 0
\end{align*}
Ce qui nous donne:
\begin{align*}
B &= \left\{ 1;2 \right\} \\
B^{-1} &= \left\{ 1;0.5 \right\}
\end{align*}
\item Nous n'avons pas la stationnarité, car $\alpha$ n'est pas à l'intérieur du cercle unité
\item
\begin{align*}
y_2 &= 1.5 y_1 - 0.5y_0 + \epsilon_2 \\
y_3 &= 1.5 y_2 - 0.5y_1 + \epsilon_3 \\
&= \epsilon_3 + 1.5 \epsilon_2 + 1.75y_1 - 0.75 y_0 \\
y_4 &= 1.5 y_3 - 0.5y_2 + \epsilon_4 \\
&= \epsilon_4 + 1.5 \epsilon_3 + 1.75 \epsilon_2 + 1.875 y_1 - 0.875 y_0
\end{align*}
On peut observer un motif répétifif, que l'on représente sous la forme
\begin{align*}
y_t &= \sum_{i=0}^{t-2} \alpha_i\epsilon_{t-i} + \alpha_{t-1}y_1 + \alpha_t y_0 \\
\end{align*}
\begin{align*}
\alpha_0 &= 1\\
\alpha_1 &= 1.5\\
\alpha_t &= 1-\alpha_{t-1}
\alpha_i &= 1.5 \alpha_{i-1} - 0.5 \alpha_{i-2}
\end{align*}
\item
\begin{align*}
y_s &= \sum_{i=0}^{s-2} \alpha_i\epsilon_{s-i} + \alpha_{s-1}y_1 + \alpha_s y_0 \\
y_{t+s} &= \sum_{i=0}^{s-2} \alpha_i\epsilon_{t+s-i} + \alpha_{s-1}y_{t+1} + \alpha_s y_t \\
E_{t+1}\left[y_{t+s}\right] &= \alpha_{s-1} y_{t+1} + \alpha_s y_t
\end{align*}
\item
\begin{align*}
E\left[y_t\right] &= \alpha_{t-1} y_1 + \alpha_t y_0 \\
E\left[y_{t+1}\right] &= \alpha_{t} y_1 + \alpha_{t+1} y_0 \\
Var\left[y_t\right] &= \left[1+\alpha_1^2 + \alpha_2^2 + \ldots + \alpha_{t-2}^2\right]\sigma^2 \\
Var\left[y_{t+1}\right] &= Var\left[y_t\right] + \alpha_{t-1}^2 \sigma^2 \\
Cov\left[y_{t},y_{t+1}\right] &= \left[\alpha_0\alpha_1 + \alpha_1\alpha_2 + \ldots + \alpha_{t-3}\alpha_{t-2}\right] \sigma^2
\end{align*}
\item
\begin{align*}
\alpha_{t} y_1 + \alpha_{t+1} y_0 \pm 1.96 * \sigma \sqrt{1+\alpha_1^2 + \alpha_2^2 + \ldots + \alpha_{t-2}^2+\alpha_{t-1}^2}
\end{align*}
\end{enumerate}
\end{solution}
\begin{solution}{2.9}
\begin{enumerate}
\item
\begin{align*}
Var[X_t] &= Var[Y_t] + Var[Z_t] \\
&= (1+\alpha^2)\sigma^2_{V} + \sigma^2_{\delta} \\
E[X_t,X_{t-1}] &= E[(v_t+\delta_t+\alpha v_{t-1})(v_{t-1}+\delta_{t-1}+\alpha v_{t-2})] \\
&= \alpha \sigma^2_{V} \\
E[X_t,X_{t-k}] &= 0, \quad\forall k \neq \{ 0,1 \} \\
&\Rightarrow \gamma_0,\gamma_1 \neq 0; \gamma_k=0 \forall k \neq \{ 0,1 \} \\
\end{align*}
Par conséquent, il s'agit d'un modèle MA(1).
\item
\begin{align*}
\gamma_0 &= (1+\theta_1^2) \sigma^2_{\epsilon} = (1+\alpha^2)\sigma^2_{V} + \sigma^2_{\delta} \\
\gamma_1 &= \theta\sigma^2_{\epsilon} = \alpha\sigma^2_{V}
\end{align*}
On résous ce système d'équations numériquement:
\begin{Schunk}
\begin{Sinput}
> fun1 <- function(par,alpha,sV,sdelta)
+ {
+ sqrt(((1+alpha^2)*sV+sdelta-(1+par[1]^2)*par[2])^2
+ +(alpha*sV-par[1]*par[2])^2)
+ }
> paramoptimaux1 <- round(optim(c(0.4,0.04),fun1,,0.5,0.04,0.01)$par,5)
\end{Sinput}
\end{Schunk}
On a donc que $\theta$ = 0.38197 et $\sigma^2_{\epsilon}$ = 0.05236
On obtient le meme résultat en utilisant un logiciel de calcul symbolique comme Maxima.
\end{enumerate}
\end{solution}
\begin{solution}{2.10}
\begin{align*}
E_t[x_{t+2}] &= \phi^2 x_t = 0.4^2*1.5 = 0.24 \\
V_t[x_{t+2}] &= (1+\phi^2)\sigma^2_{\epsilon} = (1+0.4^2)*0.25 = 0.29
\end{align*}
Intervalle de confiance:
\begin{align*}
IC &= E_t[x_{t+2}] \pm \Phi^{-1}(0.975)\sqrt{V_t[x_{t+2}]} \\
&= 0.4^2*1.5 \pm 1.96 \sqrt{(1+0.4^2)*0.25} \\
&= [-0.815492;1.295492]
\end{align*}
\end{solution}

View file

@ -0,0 +1,50 @@
\section*{Chapitre \ref{chap:modeles-volatilite}}
\addcontentsline{toc}{section}{Chapitre \protect\ref{chap:modeles-volatilite}}
\begin{solution}{3.1}
On identifie d'abord la moyenne conditionnelle de $y_t$:
\begin{align}
E_{t-1}[y_t] &= E_{t-1}[a_0 + a_1 y_{t-1} + \epsilon_t] \\
&= a_0 + a_1 y_{t-1}
\end{align}
La variance conditionnelle peut alors s'obtenir en utilisant la définition habituelle:
\begin{align*}
V_{t-1}[y_t | y_{t-1}, y_{t-2}, \ldots] &= E_{t-1}[y_t - E_{t-1}[y_t]]^{2} \\
&= E_{t-1}[(a_0 + a_1 y_{t-1} + \epsilon_t)-(a_0 + a_1 y_{t-1})]^{2} \\
&= E_{t-1}[\epsilon_t]^{2} \\
&= E_{t-1}[\alpha_0 + \alpha_1\epsilon_{t-1}^2 + \alpha_2\epsilon_{t-2}^2] \\
&= \alpha_0 + \alpha_1\epsilon_{t-1}^2 + \alpha_2\epsilon_{t-2}^2
\end{align*}
La variance inconditionnelle s'obtient en trouvant la solution particulière pour $y_t$:
\begin{align*}
•y_t &= a_0 + a_1 y_{t-1} + \epsilon_t \\
&= (1+a_1)a_0 + a_1^2 y_{t-2} + a_1 \epsilon_{t-1} + \epsilon_t \\
&= \cdots \\
&= (a+a_1+a_2+a_3+\ldots)a_0 + \epsilon_t + a_1\epsilon_{t-1} + a_2\epsilon_{t-2} + \ldots \\
&= \frac{a_0}{1-a_1} + \sum_{i=0}^{\infty} a_1^{i}\epsilon_{t-i}
\end{align*}
On évalue la variance de cette dernière expression:
\begin{align*}
•Var[y_t] &= Var[\sum_{i=0}^{\infty} a_1^{i}\epsilon_{t-i}] \\
&= \sum_{i=0}^{\infty} a_1^{2i} Var[\epsilon_{t-i}] \\
&= \frac{\sigma^2}{1-a_1^2}
\end{align*}
À partir de la définition, on a que:
\begin{align*}
E[\epsilon_t^2] &= \alpha_0 + \alpha_1 E_[\epsilon_{t-1}^2] + \alpha_2 E[\epsilon_{t-2}^2].
\end{align*}
Comme la variance inconditionnelle de $\epsilon_t$ est identique à celle de $\epsilon_{t-1}$ et $\epsilon_{t-2}$, on peut affirmer que:
\begin{align*}
E[\epsilon_t^2] &= \frac{\alpha_0}{1-\alpha_1-\alpha_2} \\
&= \sigma^2.
\end{align*}
On obtient donc que la variance inconditionnelle de $y_t$ est
\begin{align*}
•Var[y_t] &= \frac{\alpha_0}{(1-\alpha_1-\alpha_2)(1-a_1^2)}.
\end{align*}
\end{solution}

11
solutions.tex Normal file
View file

@ -0,0 +1,11 @@
\chapter{Solutions}
\label{chap:solutions}
\input{solutions-methodes-lissage}
\input{solutions-modeles-classiques}
\input{solutions-modeles-volatilite}
%%% Local Variables:
%%% mode: latex
%%% TeX-master: "exercices_series_chrono"
%%% End:

24
vgmath.sty Normal file
View file

@ -0,0 +1,24 @@
\NeedsTeXFormat{LaTeX2e}
\ProvidesPackage{vgmath}[2004/01/21 (Dr. Vincent Goulet)]
\newcommand{\pt}{{\scriptscriptstyle \Sigma}}
\newcommand{\pro}[2]{\mathrm{pro}(#1 | \mathcal{L}\{#2\})}
\newcommand{\prom}[2]{\mathrm{pro}(#1 | \mathcal{L}_{#1}\{#2\})}
\newcommand{\spn}[1]{\overline{\mathrm{sp}}\{ #1 \}}
\renewcommand{\L}[1]{\mathcal{L}\{ #1 \}}
\newcommand{\Lm}[2]{\mathcal{L}_{#1}\{ #2 \}}
\newcommand{\pscal}[2]{\langle #1, #2 \rangle}
\newcommand{\abs}[1]{\lvert #1 \rvert}
\newcommand{\norme}[1]{\lVert #1 \rVert}
\newcommand{\Esp}[1]{E\! \left[ #1 \right]}
\newcommand{\esp}[1]{E [ #1 ]}
\newcommand{\Var}[1]{\mathrm{Var}\! \left[ #1 \right]}
\newcommand{\var}[1]{\mathrm{Var} [ #1 ]}
\newcommand{\Varmat}[1]{\mathbf{V}\! \left[ #1 \right]}
\newcommand{\varmat}[1]{\mathbf{V} [ #1 ]}
\newcommand{\Cov}{\mathrm{Cov}}
\newcommand{\D}{\displaystyle}
\newcommand{\mat}[1]{\mathbf{#1}}
\newcommand{\diag}{\mathrm{diag}}
\endinput

11
vgsets.sty Normal file
View file

@ -0,0 +1,11 @@
\NeedsTeXFormat{LaTeX2e}
\ProvidesPackage{vgsets}[2004/01/21 (Dr. Vincent Goulet)]
\RequirePackage{amsfonts}
\newcommand{\R}{\mathbb{R}}
\newcommand{\N}{\mathbb{N}}
\newcommand{\Z}{\mathbb{Z}}
\newcommand{\Q}{\mathbb{Q}}
\newcommand{\C}{\mathbb{C}}
\endinput